You are on page 1of 29

MINISTRY OF PUBLIC HEALTH OF UKRAINE

Department of human resources policy, education and science

Testing Board

Student ID Last name

Variant ___________________

Test items for licensing examination

Krok 2
MEDICINE
General Instruction
Every one of these numbered questions or unfinished statements in
this chapter corresponds to answers or statements endings. Choose the
answer (finished statements) that fits best and fill in the circle with the
corresponding Latin letter on the answer sheet.

Authors of items: Andrusiak O.V., Andrusha A.B., Artyshchenko V.A., Baburіna O.A., Babyn Yu.F.,
Badogіna L.P., Balabuieva S.V., Berezov V.M., Boiko M.І., Bondarenko V.V., Borovkova S.O.,
Borzova O.Yu., Bukhtieieva E.R., Buriak V.M., Burka O.A., Butvyn І.M., Bіlenko O.A., Bіlonko O.F.,
Bіlyk O.V., Bіlyk V.D., Chekanov S.L., Chonka І.І., Chuiko A.P., Demchenko T.V., Derkach V.G.,
Desiatska Yu.V., Detsyk O.Z., Dobrovolska L.M., Dziuba G.A., Dzіs N.P., Emіralіieva Z.R., Galagan S.І.,
Genyk N.І., Gerasymenko O.І., Golubovska N.M., Goriachev V.V., Grygorova І.A., Gryshchenko V.I.,
Gutsalenko O.O., Іvaniuta S.O., Kabeliuzhenko S.B., Karlіichuk O.O., Kelmanska S.І., Kolesnyk O.M.,
Kolesnіkova O.V., Koltsova N.І., Kondratiev V.O., Konopkіna L.І., Korobko O.A., Koval І.І.,
Kovaliova O.M., Kovtunenko R.V., Kozhushko M.Yu., Kravchenko T.Yu., Kriachkova L.V., Krut Yu.Ya.,
Krylova V.Yu., Kudrevych O.M., Kudria V.I., Kudіievskyi A.V., Kutuzov І.M., Kuzmenko S.A.,
Kvasnytska O.B., Kyrylenko V.A., Lakusta N.M., Lazar A.P., Lotysh N.G., Lymar L.Ye., Lysenko D.A.,
Maliovanyi V.V., Martychnuk O.A., Marushko Iu.V., Melnychuk L.V., Miakіnkova L.O., Moroz І.M.,
Mostovoi V.P., Mostovyi Yu.M., Murtazіn L.M., Mysak A.І., Mіtiunіna N.І., Mіziuk V.V., Nykoniuk T.R.,
Nіshkumai O.І., Nіtsovych R.M., Okhrіmenko G.І., Olіfierovska R.P., Orel Yu.G., Orlova N.M.,
Ostropolets M.S., Parashchuk Iu.S., Paustovskyi Yu.O., Plastunov B.A., Pogorelov O.V., Pokydko M.І.,
Prokhoda A.F., Prokhorov Ye.V., Prus L.O., Pshuk Ya.І., Pіsotska L.A., Radchenko O.M., Reva V.B.,
Riapolova T.L., Romanenko V.N., Rudenko M.M., Sander S.V., Selska O.V., Semenukha K.V., Sergeta І.V.,
Shehovtseva T.G., Sheremeta R.Z., Shiutiev M.M., Shkurba A.V., Shkvarkovskyi І.V., Shumko G.І.,
Shusterman T.Y., Shutka V.Ya., Shvygar L.V., Sierkova V.K., Sliepіchko Yu.M., Slyva V.І., Smoliak L.L.,
Snіzhko S.S., Soldak І.І., Sopov G.O., Sotnik Yu.P., Spіrіna І.D., Stefaniuk V.D., Stepanov Yu.І.,
Strechen S.B., Svyrydova V.V., Sytnyk M.M., Sіnchuk N.І., Talalaienko Yu.O., Tarallo V.L.,
Teliushchenko O.D., Tkach V.Ye., Tkachenko M.M., Tomash O.V., Tomkіv V.M., Trefanenko І.V.,
Tsertіi V.P., Tsyvenko O.І., Uryvaieva M.K., Vanhanen N.V., Vereshchagіna N.Ia., Volianska A.G.,
Voloshyn O.M., Vorokhta Yu.M., Yakovenko І.K., Yartseva D.O., Yasnіtska M.Ya., Yermachenko T.P.,
Yevtushenko V.V., Yudіna T.V., Yurchenko І.V., Zadorozhnyi V.V., Zaiats Z.Ye., Zaitsev V.S.,
Zeliak M.V., Zhelіba M.D., Zorіna S.M., Zvierieva O.A. and Committees of professional expertise.

Item reviewers. Agafonova O.O., Anisimov Ye.M., Bab’yak T.Ye., Chursina T.Ya., Dyndar O.A.,
Grygorov Yu.B., Grynzovs’ky A.M., Gubka V.O., Gutsalenko O.O., Kalinina S.Yu., Karapetyan K.G.,
Khrapach V.V., Kolesnyk O.M., Kolosovych I.V., Kondratyuk V.Ye., Kopcha V.S., Kravchenko O.V.,
Kutovy O.B., Kuz’mina I.Yu., Malanchuk L.M., Martynyuk L.P., Mishchenko V.P., Muravs’ka O.M.,
Petrushenko V.V., Prokhorova M.P., Pryshlyak O.Ya., Puzanova O.G., Pyptyuk O.V., Shestakova I.V.,
Shevtsova T.I., Stovban I.V., Tsvirenko S.M., Usenko S.G., Vakal’uk I.P., Volyans’ka A.G.

The book includes test items for use at licensing integrated examination “Krok 2. Medicine” and
further use in teaching.

The book has been developed for students of medical, pediatric and medical-and-prophylactic
faculties and academic staff of higher medical educational establishments.

Approved by Ministry of Public Health of Ukraine as examination and teaching


publication based on expert conclusions (Orders of MPH of Ukraine of
14.08.1998 №251, of 27.12.1999 №303, of 16.10.2002 №374, of 29.05.2003 №233).

© Copyright Testing Board.


Крок 2 Medicine (англомовний варiант, iноземнi студенти) 2016 рiк 1

1. You are a doctor on duty. A patient after Common blood analysis: erythrocytes - 2,8
a successful resuscitation (drowning) was deli- Т/l, Hb- 80 g/l, color index - 0,78, anisocytosis,
vered to an admission room. BP is 90/60 mm poikilocythemia, serum iron - 10 mcmol/l.
Hg, heart rate is 120/min., respiration rate is What diagnosis is most likely?
26/min. The patient is unconscious, pupils are
moderately dilated, general clonic and tonic A. Iron-deficiency anemia
convulsions are observed. Make the diagnosis: B. B12 -deficient anemia
C. Autoimmune hemolytic anemia
A. Postresuscitation disease D. Aplastic anemia
B. Apparent death E. Hypoplastic anemia
C. Coma of unknown origin
D. Unconsciousness 6. A 24-year-old patient visited a doctor
E. Vegetative state complaining of enlargement of his submaxi-
llary lymph nodes. Objectively: submaxi-
2. A 32-year-old welder complains of llary, axillary and inguinal lymph nodes
weakness and fever. His illness initially are enlarged. Chest X-ray shows: enlarged
presented as tonsillitis one month earlier. On lymph nodes of mediastinum. Blood test:
examination: BT- 38, 9o C , RR- 24/min., HR- erythrocytes - 3, 4 · 1012 /l, Hb- 100 g/l, blood
100/min., BP- 100/70 mm Hg, hemorrhages colour index - 0,88, platelets - 190 · 109 /l,
on the legs, enlargement of the lymph nodes. leucocytes - 7, 5·109 /l, eosinophiles - 8%, band
CBC shows Hb- 70 g/l, RBC- 2, 2 · 1012 /l, neutrophiles - 2%, segmented neutrophiles -
WBC- 3, 0 · 109 /l with 32% of blasts, 1% of 67%, lymphocytes - 23%, ESR- 22 mm/hour.
eosinophiles, 3% of bands, 36% of segments, What test must be prescribed to verify the
20% of lymphocytes, and 8% of monocytes, cause of lymphadenopathy?
ESR- 47 mm/hour. What is the cause of
anemia? A. Open biopsy of lymph nodes
B. Ultrasonography of abdominal cavity
A. Acute leukemia C. Mediastinum tomography
B. Chronic lympholeukemia D. Puncture biopsy of lymph nodes
C. Aplastic anema E. Sternal puncture
D. B12 -deficient anemia
E. Chronic hemolytic anemia 7. A 32 year old patient complains of cardiac
irregularities, dizziness, dyspnea at physical
3. A regional cardiologist is tasked with the exertion. He has never suffered from such
development of a plan for medioprophylactic condition before. Objectively: Ps- 74/min.,
measures aimed at decrease of cardiovascular rhythmic. BP- 130/80 mm Hg. Auscultati-
mortality. What measures should be planned on revealed systolic murmur above aorta,
for secondary prevention? the first heart sound was normal. ECG
showed hypertrophy of the left ventricle, si-
A. Prevention of recurrences and complicati- gns of repolarization disturbance in the I , V5
ons and V6 leads. Echocardiogram revealed that
B. Referring patients for sanatorium-and-spa interventricular septum was 2 cm. What is the
treatment most probable diagnosis?
C. Prevention of diseases
D. Referring patients for in-patient treatment A. Hypertrophic cardiomyopathy
E. Optimization of life style and living conditi- B. Aortic stenosis
ons C. Essential hypertension
D. Myocardium infarction
4. An 8-year-old boy developed a temperature E. Coarctation of aorta
of 37, 5o C two days after his recovery from
the case of URTI. He complains of suffocati- 8. A 35-year-old patient’s wound with
on, heart pain. Objectively: the skin is pale, suppurative focus was surgically cleaned.
tachycardia, the I heart sound is weakened, On the 8th day after the surgery the wound
short systolyc murmur in the 4th intercostal cleared from its purulo-necrotic content and
area near the left edge of the breastbone. granulations appeared. However, against the
What heart disorder such clincal presentati- bacground of antibacterial therapy the body
on is characteristic of? temperature keeps at 38,5-39,5o C . There are
chills, excessive sweating, euphoria, heart rate
A. Nonrheumatic myocarditis is 120/min. What complication of local pyoi-
B. Primary rheumatic carditis nflammatory process can it be?
C. Myocardiodystrophy
D. Fallot’s tetrad A. Sepsis
E. Cardiomyopathy B. Purulent absorption fever
C. Trombophlebitis
5. A woman complains of muscle weakness D. Meningitis
and general fatigue, dyspnea, vertigo, bri- E. Pneumonia
ttleness of her hair and nails, an urge to
eat chalk. Anamnesis states uterine fibroid. 9. A 60-year-old woman has been sufferi-
Крок 2 Medicine (англомовний варiант, iноземнi студенти) 2016 рiк 2

ng from arterial hypertension for 15 years. A. Tuberculous spondylitis of the thoracic


After recurrent stroke she started complaini- spine
ng of unmotivated bad mood, problems with B. Spinal tumor
attention concentration; she forgets to close C. Multiple sclerosis
the entrance door, cannot recall events of D. Metastases into the spine
the past day. Computer tomography shows E. Osteochondrosis
areas of postinfarction changes in the cortical
postfrontal areas. What is the most probable 13. A pregnant woman is 28 years old.
diagnosis? Anamnesis: accelerated labor complicated by
the II degree cervical rupture. The followi-
A. Vascular dementia ng two pregnancies resulted in spontaneous
B. Alzheimer’s disease abortions at the terms of 12 and 14 weeks.
C. Huntington’s disease On mirror examination: the uterine cervix
D. Pick’s disease is scarred from previous ruptures at 9 and 3
E. Dissociative amnesia hours, the cervical canal is gaping. On vagi-
nal examination: the cervix is 2 cm long, the
10. Clinic of a research instutute for occupati- external orifice is open 1 cm wide, the internal
onal diseases examined a worker who works orifice is half-open; the uterus is enlarged to
at a concentration plant and diagnosed him the 12th week of pregnancy, soft, mobile, pai-
with chronic dust bronchitis. The case is nless, the appendages are without changes.
investigated by a commission including the What diagnosis would you make?
representatives of: the plant, clinic, local
SES, department of Social Insurance Fund, A. Isthmico-cervical insufficiency, habitual
trade union. According to the ”regulation on noncarrying of pregnancy
investigation of. . . ”, the commission should be B. Threatened spontaneous abortion
headed by the representative of the following C. Incipient abortion, habitual noncarrying of
authority: pregnancy
D. Cervical hysteromyoma, habitual noncarryi-
A. Local SES ng of pregnancy
B. Plant E. Cervical pregnancy, 12 weeks
C. Social Insurance Fund
D. Trade union 14. A 56-year-old patient complains of pain in
E. Clinic the epigastrium after eating, eructation, loss
of appetite, slight loss of weight, fatigabili-
11. A 37-year-old woman complains of ty. The patient smokes; no excessive alcohol
headaches, nausea, vomiting, spasms. The consumption. Objectively: pale mucosa, BP-
onset of the disease occurred the day before 110/70 mm Hg. The tongue is ”lacquered”.
due to her overexposure to cold. Objecti- The abdomen is soft, sensitive in the epi-
vely: fever up to 40o C ; somnolence; rigid gastric area. Blood test: erythrocytes - 3,0 T/l,
neck; Kernig’s symptom is positive on the Hb- 110 g/l, color index - 1,1; macrocytosis;
both sides; general hyperesthesia. Blood test: leukocytes - 5,5 g/l, ESR- 13 mm/hour. On
leucocytosis, increased ESR. Cerebrospinal fibrogastroduodenoscopy: atrophy of fundic
fluid is turbid, yellow-tinted. What changes of mucosa. What pathogenesis does this disorder
the cerebrospinal fluid are most likely? have?
A. Neutrophilic pleocytosis A. Producing antibodies to parietal cells
B. Lymphocytic pleocytosis B. Н.pylori persistence
C. Blood in the cerebrospinal fluid C. Alimentary factor
D. Xanthochromia in the cerebrospinal fluid D. Chemical factor
E. Albuminocytological dissociation E. Gastropathic effect
12. A 48-year-old woman complains of pain 15. A 26-year-old secundipara at 40 weeks of
in the thoracic spine, sensitivity disorder gestation arrived at a maternity ward after
in the lower body, disrupted motor functi- the beginning of labor activity. The bursti-
on of the lower limbs, body temperature ng of waters occurred 2 hours prior. The
rise up to 37, 5o C . She has been suffering fetus was in a longitudinal lie with cephalic
from this condition for 3 years. Treatment presentation. Abdominal circumference was
by various specialists was ineffective. X-ray 100 cm, fundal height - 42 cm. Contracti-
reveals destruction of adjacent surfaces of ons occurred every 4-5 minutes and lasted
the VIII and IX vertebral bodies. In the right 25 seconds each. Internal obstetric examinati-
paravertebral area at the level of lesion there on revealed cervical effacement, opening by 4
is an additional soft tissue shadow. What di- cm. Fetal bladder was absent. Fetal head was
agnosis is most likely? pressed against the pelvic inlet. What compli-
cation arose in the childbirth?
Крок 2 Medicine (англомовний варiант, iноземнi студенти) 2016 рiк 3

A. Early amniorrhea A. Peripheral lung cancer


B. Primary uterine inertia B. Bronchiectasis
C. Secondary uterine inertia C. Pneumonia
D. Discoordinated labor D. Middle lobe syndrome
E. Clinically narrow pelvis E. Interlobular pleurisy
16. Bacterial analysis of air in a living space in 20. A 62-year-old patient has been hospitali-
winter period by means of Krotov’s apparatus zed with complaints of pain in the thorax
revealed that total number of microorganisms on the right during breathing, dyspnea, dry
in 1 m3 of air was 7200. What is the allowed cough. Ten days ago he slipped and fell hitti-
number of microorganisms for the air to be ng his right side. On examintaion: the pati-
characterized as ”pure”? ent lies on the left side. The right side of the
thorax lags during breathing. On the right
A. Up to 4500 there are crepitation and pain in the III-IV ri-
B. Up to 2500 bs. Dullness of percussion sound and sharply
C. Up to 3500 diminished breath sounds can be observed.
D. Up to 5500 On X-ray: signs of exudate, fracture of the III-
E. Up to 7500 IV ribs. On pleurocentesis: blood is detected.
Choose the further tactics:
17. A 24-year-old patient received a
puncture injury below the Poupart’s ligament A. Transfer to a thoracic surgery department
accompanied by intense arterial bleeding. The B. Prescribe conservative therapy
best method to temporarily stop the bleeding C. Recurrent pleurocentesis
in the patient would be: D. Fixed bandage of the rib cage
E. Refer to a traumatologist
A. Compression band
B. Esmarch’s tourniquet 21. A 29-year-old female patient complains of
C. Maximum limb bending dyspnea, heaviness and chest pain on the ri-
D. Compressing a blood vessel with a clamp ght, body temperature rise up to 37, 2o C . The
E. Wound suturing disease is associated with a chest trauma recei-
ved 4 days ago. Objectively: skin is pale and
18. A 42-year-old woman has been hospitali- moist. Ps- 90/min., regular. Palpation reveals
zed with complaints of intense pain attacks a dull sound on the right, auscultation reveals
in the lumbar and right iliac areas, which significantly weakened vesicular breathing.
irradiate to the vulvar lips, frequent urination, In blood: RBCs- 2, 8 · 1012 /l, colour index -
nausea. The pain onset was acute. Objecti-
vely: the abdomen is soft, moderately pai- 0,9, Hb- 100 g/l, WBCs- 8, 0 · 109 /l, ESR- 17
nful in the right subcostal area, costovertebral mm/hour. What results of diagnostic puncture
angle tenderness on the right. Common uri- of the pleural cavity can be expected?
ne analysis: specific gravity - 1016, traces of A. Haemorrhagic punctate
protein, leukocytes - 6-8 in the vision field, B. Chylous liquid
erythrocytes - 12-16 in the vision field, fresh. C. Exudate
What diagnosis can be made? D. Transudate
A. Right-sided renal colic E. Purulent punctate
B. Acute right-sided pyelonephritis 22. Caries morbidity rate is 89% among resi-
C. Acute right-sided adnexitis dents of a community. It is determined that
D. Acute cholecystitis fluorine content in water is 0,1 mg/l. What
E. Acute appendicitis preventive measures should be taken?
19. A 29-year-old patient works as a A. Water fluorination
motor mechanic. Anamnesis shows frequent B. Tooth brushing
exposure to cold, exacerbation of chronic C. Fluorine inhalations
bronchitis attended by cough with relati- D. Sealant application
vly small amount of mucopurulent sputum, E. Introduce more vegetables to the diet
subfebrility, sometimes joined by hemoptysis
and pain in the right side of chest. Breathing 23. Examination of a group of persons living
is vesicular. X-ray shows shadows and sharp on the same territory revealed the following
decrease in size of the lower lobe distinctly vi- common symptoms: dark-yellow pigmentati-
sible on the X-ray image as a streak 2-3 cm wi- on of the tooth enamel, diffuse osteoporosis
de situated at the angle from lung root to the of bone apparatus, ossification of ligaments
frontal costodiaphragmatic recess. The most and joints, functional disorders of the central
likely diagnosis is: nervous system. This condition may be caused
by the excessive concentration of the followi-
ng microelement in food or drinking water:
Крок 2 Medicine (англомовний варiант, iноземнi студенти) 2016 рiк 4

A. Fluorine aside from tachycardia (102/min.) no other


B. Copper changes. On ECG: pathologic wave Q in I,
C. Nickel аVL, QS in V1, V2, V3 leads and ’domed’ ST
D. Iodine elevation with negative T. What diagnosis is
E. Cesium most likely?
24. In a pre-school educational establishment A. Acute left ventricular anterior myocardial
the menu consists of the following dishes: milk infarction
porridge from buckwheat, pasta with minced B. Variant angina pectoris
meat, cucumber salad, kissel (thin berry jelly), C. Aortic dissection
rye bread. What dish should be excluded from D. Acute left ventricular posterior myocardial
the menu? infarction
E. Exudative pericarditis
A. Pasta with minced meat
B. Milk porridge from buckwheat 29. A 48-year-old woman has thermal burns
C. Kissel (thin berry jelly) of both hands. The epidermis of the palms
D. Rye bread and backs of her hands is exfoliating, and bli-
E. Cucumber salad sters filled with serous liquid are forming. The
forearms are intact. What diagnosis is most
25. An infant has been born at the 41st week likely?
of gestation. The pregnancy was complicated
with severe gestosis of the second semester. A. 2-3A degree thermal burn
The weight of the baby is 2400 g, the height is B. 4 degree thermal burn
50 cm. Objectively: the skin is flabby, the layer C. 1 degree thermal burn
of subcutaneous fat is thin, hypomyotonia, D. 3B degree thermal burn
neonatal reflexes are weak. The internal E. 1-2 degree thermal burn
organs are without pathologic changes. This
newborn can be estimated as a: 30. A 39-year-old patient complains of morni-
ng headache, appetite loss, nausea, morning
A. Full-term infant with prenatal growth vomiting, periodic nasal haemorrhages. The
retardation patient had a case of acute glomerulonephri-
B. Premature infant tis at the age of 15. Examination revealed rise
C. Immature infant of arterial pressure up to 220/130 mm Hg, skin
D. Postmature infant haemorrhages on his arms and legs, pallor of
E. Full-term infant with normal body weight skin and mucous membranes. What biochemi-
cal parameter is most important for making
26. A patient suffering from infiltrati- diagnosis in this case?
ve pulmonary tuberculosis was prescribed
streptomycin, rifampicin, isoniazid, pyrazi- A. Blood creatinine
namide, vitamin C. One month after the begi- B. Blood bilirubin
nning of the treatment the patient started C. Blood sodium
complaining of reduced hearing and tinnitus. D. Uric acid
What drug has such a side effect? E. Fibrinogen
A. Streptomycin 31. A 30-year-old patient, who has
B. Isoniazid been suffering from headaches, suddenly
C. Rifampicin developed extreme headache after lifting a
D. Pyrazinamide heavy load, as if he had been hit over the
E. Vitamin C head. Nausea, vomiting, and slight dizzi-
ness are observed. In a day he developed
27. A woman has developed sudden thoracic pronounced meningeal syndrome and body
pain on the right with expectoration of pi- temperature up to 37, 6o C . A doctor suspects
nk sputum and body temperature rise up to subarachnoid hemorrhage. What additional
37, 7o C on the 4th day after the surgery for examination is necessary to confirm this di-
cystoma of the right ovary. On lung examinati- agnosis?
on: dullness of the lung sound on the lower
right is observed. Isolated moist crackles can A. Lumbar puncture with investigation of the
be auscultated in the same area. What compli- spinal fluid
cation is the most likely? B. Skull X-ray
C. Computed tomography of the brain
A. Pulmonary infarction D. Rheoencephalography
B. Pneumonia E. Angiography of the brain vessels
C. Pulmonary abscess
D. Exudative pleurisy 32. A worker of a blowing shop complains of
E. Pneumothorax headache, irritability, sight impairment - he
sees everything as if through a ”net”. Objecti-
28. A 58-year-old patient was delivered to an vely: hyperemic sclera, thickened cornea,
admission room with complaints of pain in the decreased opacity of pupils, visual acuity is
thorax on the left. On clinical examination:
Крок 2 Medicine (англомовний варiант, iноземнi студенти) 2016 рiк 5

0,8 in the left eye, 0,7 in the right eye. The operator?
worker uses no means of personal protection.
What diagnosis is most likely? A. Photoelectric ophthalmia
B. Heatstroke
A. Cataract C. Vegetative-vascular dystonia
B. Conjunctivitis D. Chronic overheating
C. Keratitis E. Pneumoconiosis
D. Blepharospasm
E. Progressive myopia 37. A 48-year-old patient was found to have
diffuse enlargement of the thyroid gland,
33. A 45-year-old woman is undergoing exophthalmia, weight loss of 4 kg in 2 months,
treatment for active rheumatism, combined sweating. Objectively: HR- 105/min., BP-
mitral valve failure. During her morning 140/70 mm Hg. Defecation act is normal.
procedures she suddenly sensed pain in the What kind of therapy is recommended in this
left hand, which was followed by numbness. case?
Pain and numbness continued to aggravate.
Objectively: the skin of the left hand is pale A. Mercazolil
and comparatively cold. Pulse in the hand B. Radioiodine
arteries is absent along the whole length. C. Propranolol
What treatment tactics is most efficient? D. Lugol’s solution
E. Thyroxine
A. Urgent embolectomy
B. Prescription of fibrinolytics and anti- 38. A woman addressed a doctor with
coagulants complaints of increased body temperature up
C. Prescription of antibiotics and antii- to 37, 8o C and moderately sore throat for the
nflammatory agents last 3 days. Objectively: mandibular lymph
D. Cardiac catheterization nodes are enlarged up to 3 cm. Palatine tonsils
E. Urgent thrombintimectomy are hypertrophied, covered with grey coating
that spreads to the uvula and anterior pillars
34. A 10-year-old boy had a case of viral of the fauces. What diagnosis is most likely?
hepatitis type B four years ago. Currently the
assumption was made about the formation of A. Oropharyngeal diphtheria
hepatic cirrhosis in the patient. What additi- B. Infectious mononucleosis
onal investigation can clarify the diagnosis? C. Pseudomembranous (Vincent’s) tonsillitis
D. Agranulocytosis
A. Renal needle biopsy E. Oropharyngeal candidiasis
B. Proteinogram
C. Echocholecystography 39. A 48-year-old man complains of constant
D. Markers of viral hepatitis type B pain in the upper abdomen, predominantly
E. Transaminase level measurement on the left, which aggravates after eating,
diarrhea, loss of weight. The patient has
35. A 40-year-old patient has acute onset alcohol use disorder. Two years ago he had
of disease caused by overexposure to cold. a case of acute pancreatitis. Blood amylase
Temperature has increased up to 39o C . is 4 g/hour·l. Feces analysis: steatorrhea,
Foul-smelling sputum is expectorated duri- creatorrhea. Blood sugar is 6,0 mmol/l. What
ng coughig. Various moist crackles can be treatment should be prescribed?
auscultated above the 3rd segment on the ri-
ght. Blood test: leukocytes - 15, 0 · 109 /l, stab A. Panzinorm forte (Pancreatin)
neutrophils - 12%, ESR- 52 mm/hour. On X- B. Insulin
ray: in the 3rd segment on the right there is a C. Gastrozepin (Pirenzepine)
focus of shadow 3 cm in diameter, low density, D. Contrykal (Aprotinin)
with fuzzy smooth margins and a clearing in E. No-Spa (Drotaverine)
its center. What disease is most likely in the
given case? 40. In 10 hours after eating canned
mushrooms a 27-year-old patient has
A. Pneumonia complicated by an abscess developed diplopia, bilateral ptosis, di-
B. Infiltrative tuberculosis srupted swallowing, shallow breathing with
C. Peripheral pulmonary cancer respiratory rate 40/min., muscle weakness,
D. Cystic echinococcosis enteroparesis. What measure should be taken
E. Pulmonary cyst first?
36. An electro-gas welding operator worki- A. Intubation of the trachea for artificial
ng at a machine workshop performs welding respiration
and cutting of metal, which is accompanied B. Gastrointestinal lavage
by intense UV-radiation. His welding station C. Introduction of antibotulinic serum
is equipped with efficient mechanical venti- D. Introduction of glucocorticosteroids
lation. What occupational disease is most E. Intravenous detoxication therapy
likely to develop in an electro-gas welding
41. A 32-year-old patient complains of
Крок 2 Medicine (англомовний варiант, iноземнi студенти) 2016 рiк 6

reddening, burning, and sensation of a foreign labor was diagnosed with cephalohematoma.
body in the right eye. The disease is acute. On On the 2nd day of life the child developed
examination: visual acuity of the both eyes jaundice; on the 3rd day of life there appeared
is 1,0. In the right eye there are hyperemy neurological changes: nystagmus, Graefe
and swelling of the conjunctiva, superficial syndrome. Urine is yellow, feces are golden-
injection. There is purulent discharge in the yellow. The mother’s blood group is А (II)
conjunctival sac. The cornea is clear. The color Rh− , the child’s - А (II) Rh+ . On the 3rd
and pattern of the iris are uncanged, the pupil day the results of the child’s blood test are as
is mobile. What diagnosis is most likely? follows: Hb- 200 g/l, erythrocytes - 6, 1 · 1012 /l,
A. Acute conjunctivitis blood bilirubin - 58 mcmol/l due to the
B. Acute iridocyclitis presence of its unconjugated fraction, Ht- 0,57.
C. Acute attack of glaucoma In this case the jaundice is caused by:
D. Foreign body of the cornea A. Craniocerebral birth injury
E. Acute dacryocystitis B. Physiologic jaundice
42. A patient is on the sick leave for 4 months C. Hemolytic disease of newborn
continuously from the date of injury. The D. Atresia of bile passages
treatment is going to last for another 1-2 E. Fetal hepatitis
months. Who has the right to extend the 46. On the 4th day after recovering from a
duration of medical certificate for this pati- cold a patient was hospitalized with complai-
ent? nts of solitary spittings of mucoid sputum.
A. Medical advisory commission after medico- On the 2nd day there was a single discharge
social expert commission examination of about 250 ml of purulent blood-streaked
B. Medical superintendent sputum. Objectively: the patient’s conditi-
C. Medical advisory commission after inpati- on is moderately severe. Respiratory rate -
ent treatment 28-30/min., Ps- 96/min., BP- 110/70 mm Hg.
D. District doctor upon agreement with a Respiration above the left lung is vesicular,
department chief weak above the right lung. There are vari-
E. Medico-social expert commission ous moist crackles above the lower lobe and
amphoric breath near the angle of scapula.
43. Monthly dysentery morbidity in the region What is the most likely diagnosis?
given in absolute figures is as follows: January
- 6; February - 9; March - 11; April - 10; May - A. Acute pulmonary abscess
16; June - 23; July - 19; August - 33; September B. Exudative pleuritis
- 58; October - 19; November - 11; December C. Acute focal pneumonia
- 5. Annual total is 220 cases. What graphic D. Pleural empyema
presentation would provide the best visual for E. Pyopneumothorax
monthly deviations of dysentery morbidity 47. A 46-year-old patient with temporari-
from the average? ly undetermined diagnosis was prescribed
A. Radar chart pleurocentesis based on the results of the X-
B. Map ray. The puncture yielded 1000 ml of a liquid
C. Cartogram with the following properties: clear, specific
D. Pie chart gravity - 1,010, protein content - 1%, Rivalta’s
E. Bar chart test is negative, erythrocytes - 2-3 in the field
of vision. What disorder are these pathologic
44. A 30-year-old woman complains of pain changes characteristic of?
in the heart area (”aching, piercing pain”) A. Cardiac failure
that arises primarily in the morning hours in B. Pleuropneumonia
autumn and spring. Pain irradiates into the C. Pleural mesothelioma
neck, back, abdomen and is attended by rapid D. Pulmonary tuberculosis
heart rate and low vital tonus. This condition E. Pulmonary cancer
occurs independently from physical exerti-
on. In the evening her condition improves. 48. A 14-year-old girl has been presenting wi-
Somatic and neurologic state and ECG have th irritability and tearfulness for about a year.
no pathologies. What pathology is likely to A year ago she was also found to have diffuse
result in such clinical presentation? enlargement of the thyroid gland (II grade).
A. Somatized depression This condition was regarded as a pubertal
B. Rest angina pectoris manifestation, the girl did not undergo any
C. Pseudoneurotic schizophrenia treatment. The girl’s irritability gradually gave
D. Somatoform autonomic dysfunction place to a complete apathy. The girl developed
E. Hypochondriacal depression puffy face, soft tissues pastosity, bradycardia,
constipations. Skin pallor and gland density
45. A full term baby born from the 1st progressed, the skin developed a waxen hue.
noncomplicated pregnancy with complicated What disease can be suspected?
Крок 2 Medicine (англомовний варiант, iноземнi студенти) 2016 рiк 7

A. Autoimmune thyroiditis 53. A 56-year-old patient with diffuse toxic


B. Diffuse toxic goiter goiter has ciliary arrhythmia, heart rate is 110-
C. Thyroid carcinoma 120/min., arterial hypertension, BP is 165/90
D. Subacute thyroiditis mm Hg. What drug besides Mercazolil (Thi-
E. Juvenile basophilism amazole) should be prescribed in this case?
49. In a maternity hospital a newborn had A. Propranolol
been presenting with cough attacks after eati- B. Radioactive iodine
ng. The child was discharged from the hospital C. Novocainamide (procainamide)
on the 18th day due to a case of pneumonia. D. Verapamil
During the further 1,5 months the child had E. Corinfar
2 cases of pneumonia. Periodically there are
cough attacks after eating, especially if the 54. A patient has been provisionally di-
child lies on the left side. Objectively: the II agnosed with pheochromocytoma at the stage
degree hypotrophy, isolated moist crackles, of intermission. BP is within norm, there is
dyspnea. Stool and diuresis are not disrupted. a tendency towards tachycardia. No urine
What diagnosis is most likely? pathologies. The decision has been made to
perform a provocative test with histamine.
A. Tracheoesophageal fistula What drug should be kept close at hand for
B. Mucoviscidosis emergency aid in case of positive test result?
C. Posthypoxic encephalopathy
D. Hernia of the esophageal opening A. Phentolamine
E. Tracheobronchomalacia B. Pipolphen
C. Nifedipine
50. A 26-year-old woman, who gave bi- D. Mesaton (Phenylephrine)
rth 7 months ago, has been suffering from E. Prednisolone
nausea, morning sickness, somnolence for
the last 2 weeks. The patient breasfeeds; no 55. A 54-year-old patient complains of
menstruation. She has been using no means of weakness, weight loss despite the unchanged
contraception. What method would be most appetite, frequent urination, skin itch for six
efficient in clarification of the diagnosis? months. Some time ago the patient underwent
treatment for furunculosis. She has not been
A. Ultrasound examined recently. Objectively: malnutriti-
B. Small pelvis radiography on, dry skin with scratch marks. Small lymph
C. Palpation of mammary glands and squeezi- nodes can be palpated in the axillary regions.
ng out colostrum Changes in the internal organs are absenr.
D. Bimanual abdominovaginal examination What test must be performed in the first
E. Mirror examination place?
51. A 2-year-old girl has a medical history A. Fasting blood sugar
of recurrent obstructive pneumonia. In the B. Complete blood count
lungs various moist and dry crackles can C. Endoscopy of stomach
be auscultated, breath sounds are dimini- D. Lymph node biopsy
shed. Sputum is thick, viscous and difficult E. Blood sterility testing
to expectorate. Drumstick fingers and physi-
cal developmental retardation are observed. 56. A child is 1 month 10 days old. ”Gushing”
What preliminary diagnosis can be made? vomiting has been observed since the age of 3
weeks. The vomit volume exceeds the volume
A. Pulmonary mucoviscidosis of the previous feeding. Objectively: the chi-
B. Recurrent bronchitis ld is inert. Skin elasticity and tissue turgor
C. Bronchial asthma are decreased. Hour-glass deformity sign is
D. Congenital polycystic lungs positive. The preliminary diagnosis is pyloric
E. Pulmonary tuberculosis stenosis. What treatment tactics should be
52. After a case of purulent otitis a 1-year-old chosen?
boy has developed pains in the upper third of A. Surgery
the left thigh, body temperature up to 39o C .
Objectively: swelling of the thigh in its upper B. Prescription of Cerucal (Metoclopramide)
third and smoothed out inguinal fold. The li- C. Atropinization
mb is in semiflexed position. Active and passi- D. Internal administration of Novocaine
ve movements are impossible due to severe E. Prescription of Pipolphen
pain. What diagnosis is most likely? 57. A 46-year-old woman complains of pain
A. Acute hematogenous osteomyelitis attacks in the right lumbar area, which irradi-
B. Acute coxitis ate to the lower abdomen, and nausea.
C. Intermuscular phlegmon This kind of pain attacks has never been
D. Osteosarcoma detected in the patient before. Survey X-
E. Brodie’s abscess ray of the abdominal cavity reveals no
pathologic shadows. Ultrasonic scan detects
Крок 2 Medicine (англомовний варiант, iноземнi студенти) 2016 рiк 8

a hyperechogenic growth 1,5 cm in diameter, 27 10 . What drug would be most efficient in
which reflects sound wave, in the enlarged ri- the treatment of this patient?
ght renal pelvis. What diagnosis is most likely?
A. Cryoprecipitate
A. Renal calculus B. Calcium chloride
B. Benign renal tumor C. Concentrated red cells
C. Renal cyst D. Aminocapronic acid
D. Renal tuberculosis E. Vicasol (Menadione)
E. Malignant renal tumor
62. A 67-year-old man complains of dyspnea
58. Several hours before, a 28-year-old pati- on exertion, attacks of retrosternal pain, di-
ent suddenly developed acute headache and zziness. He has no history of rheumatism.
repeated vomiting, then lost consciousness. Objectively: pale skin, acrocyanosis. There are
Objectively: focal neurological symptoms crackles in the lower lungs. There is systolic
were not found. Pronounced meningeal thrill in the II intercostal space on the ri-
symptoms were revealed. BP- 120/80 mm ght, coarse systolic murmur conducted to the
Hg. According to clinical and liquorologi- vessels of neck. BP- 130/90 mm Hg, heart rate
cal findings the patient was diagnosed wi- - 90/min., regular rhythm. The liver extends 5
th subarachnoid haemorrhage. After admi- cm from under the edge of costal arch, shin
nistration of dehydrants the patient’s condi- edemas are present. Specify the suspected
tion somewhat improved. What is the main valvular defect:
component of further emergency care?
A. Aortic stenosis
A. Coagulants B. Pulmonary artery stenosis
B. Anticoagulants C. Mitral insufficiency
C. Antiaggregants D. Ventricular septal defect
D. Fibrinolytics E. Tricuspid regurgitation
E. Corticosteroids
63. A 24-year-old female teacher complai-
59. A 24-year-old man has developed increasi- ns of dizziness and heart pain irradiating
ng headaches, vertigo, diplopia, paresis of the to the left nipple. Pain is not associated
facial muscles on the right, choking during with physical activity and cannot be reli-
swallowing. The signs appeared on the 5th eved by nitroglycerin, it abates after taki-
day of respiratory disorder. He was diagnosed ng Valocordin and lasts an hour or more.
with acute viral encephalitis. Determine the The patient has a nearly 2-year history of
main direction of emergency therapy: this disease. Objectively: Ps- 76/min., BP-
110/70 mm Hg. Heart borders are normal,
A. Zovirax (Aciclovir) heart sounds are clear. The ECG shows respi-
B. Glucocorticoids ratory arrhythmia. Radiograph of the cervi-
C. Ceftriaxone cothoracic spine reveals no pathology. Lungs,
D. Lasix (Furosemide) abdomen are unremarkable. What changes in
E. Neohaemodes blood formula can be expected?
60. A 23-year-old woman, who works as a A. No changes
milk and dairy inspector, after the miscarri- B. Leukocytosis
age suffers from high fever up to 38, 6o C , C. Thrombocytopenia
recurring chills, excessive sweating. Objecti- D. Leukemic hiatus
vely: polyadenitis, pain in the lumbosacral E. Increased ESR
spine, swollen left knee joint, enlarged liver
and spleen. What diagnosis is most likely? 64. A 51-year-old female patient complains of
frequent defecation and liquid blood-streaked
A. Brucellosis stools with mucus admixtures, diffuse pain in
B. Sepsis the inferolateral abdomen, 6 kg weight loss
C. Toxoplasmosis within the previous month. Objectively: body
D. Polyarticular rheumatoid arthritis temperature - 37, 4o C , malnutrition, skin is
E. Yersiniosis pale and dry. Abdomen is soft, sigmoid is pai-
nful and spasmodic, makes a rumbling sound.
61. A 16-year-old adolescent has been hospi- Liver is dense, painful, extends 3 cm below
talized with complaints of unceasing nasal the costal margin. What is the most likely di-
hemorrhage and unbearable pain in his ri- agnosis?
ght elbow joint. Objectively: the large joint
is enlarged and defigurated, the skin over A. Non-specific ulcerative colitis
the joint is hyperemic. Arthropathy signs can B. Bacillary dysentery
be observed in the other joints. Ps- 90/min. C. Sprue
Blood test: erythrocytes - 3, 9 · 1012 /l, Нb- 130 D. Intestinal enzymopathy
g/l, color index - 1,0, leukocytes - 5, 6 · 109 /l, E. Helminthic invasion
platelets - 220 · 109 /l, ESR- 6 mm/hour.
65. Chief physician of a polyclinic charged a
Lee-White coagulation time: start- 24 , end-
Крок 2 Medicine (англомовний варiант, iноземнi студенти) 2016 рiк 9

district doctor with a task to determine the A. Platelets


pathological prevalence of disease N in his B. Reticulocytes
district. What document allows to estimate C. Clotting time
the disease prevalence in the population of a D. Osmotic resistance of erythrocytes
medical district? E. Fibrinogen

A. Prophylactic examinations register 69. A 49-year-old patient complains of


B. Statistic coupons (+) swallowing disorder that intensifies duri-
C. Statistic coupons (-) ng eating solid food, hiccups, hoarse voice,
D. Statistic coupons (+) and (-) nausea, regurgitation, significant weight loss
E. Vouchers for medical appointments (15 kg within 2,5 months). Objectively: body
weight is reduced; the skin is pale and dry;
66. A 32-year-old woman complains of di- vesicular respiration; heart sounds are suffi-
zziness, headache, palpitation, tremor. For ciently sonorous; heart rate is rhythmic. The
the last several months she has been under abdomen is soft, no pain on palpation. The
outpatient observation for increased arteri- liver is not enlarged. What investigation is
al pressure. Since recently such attacks have most necessary for making the diagnosis in
become more frequent and severe. Objecti- this case?
vely: the skin is covered with clammy sweat,
tremor of the extremities is present. HR- A. Esophagoduodenoscopy with biopsy
110/min., BP- 220/140 mm Hg. Heart sounds B. Clinical blood analysis
are muffled. Blood test results: WBCs- 9, 8 · C. X-ray of the gastrointestinal tract
D. X-ray in the Trendelenburg position
109 /l, ESR- 22 mm/hour. Blood glucose - 9,8 E. Investigation of gastric secretion
millimole/l. What disease is the most likely
cause of this crisis? 70. A 9-month-old child presents with fever,
cough, dyspnea. The symptoms appeared
A. Pheochromocytoma
5 days ago after a contact with a person
B. Essential hypertension suffering from URTI. Objectively: the child
C. Preeclampsia is in grave condition. Temperature of 38o C ,
D. Primary hyperaldosteronism
cyanosis of nasolabial triangle is present.
E. Diabetic glomerulosclerosis Respiration rate - 54/min., nasal flaring duri-
67. A 45-year-old patient complains of pain ng breathing. There was percussion dullness
in the epigastric region, left subcostal area, on the right below the scapula angle, and
abdominal distension, diarrhea, loss of wei- tympanic sound over the rest of lungs.
ght. He has been suffering from this condition Auscultation revealed bilateral fine moist
for 5 years. Objectively: tongue is moist with crackles predominating on the right. What is
white coating near the root; deep palpation of the most likely diagnosis?
abdomen reveals slight pain in the epigastric
A. Acute pneumonia
region and Мауо-Robson’s point. Liver is pai- B. URTI
nless and protrudes 1 cm from the costal arch. C. Acute laryngotracheitis
Spleen cannot be palpated. What disease can
D. Acute bronchitis
be primarily suspected? E. Acute bronchiolitis
A. Chronic pancreatitis
71. A 50-year-old man, who works as a poli-
B. Atrophic gastritis sher at a combine-building factory, addressed
C. Peptic stomach ulcer the factory’s sectorial doctor with complai-
D. Chronic cholecystitis
nts of general fatigue, sensations of numbness
E. Chronic enteritis and pain in his fingers. Objectively: the skin
68. A 58-year-old woman complains of of his fingers is pale. Reaction to pain, tactile
spontaneous bruises, weakness, bleeding and thermal stimuli was revealed to be slightly
gums, dizziness. Objectively: the mucous disrupted. No disruptions can be observed wi-
membranes and skin are pale with numerous thin the other organs and systems. What di-
hemorrhages of various time of origin. Lymph sorder is most likely?
nodes are not enlarged. Heart rate - 100/min., A. Pneumatic hammer disease
BP- 110/70 mm Hg. There are no changes
B. Multiple neuritis
of internal organs. Blood test results: RBC- C. Raynaud’s disease
3, 0 · 1012 /l, Нb- 92 g/l, colour index - 0,9, D. Syringomyelia
anisocytosis, poikilocytosis, WBC - 10 · 109 /l, E. Deforming arthrosis
eosinophiles - 2%, stab neutrophiles - 12%,
segmented neutrophiles - 68%, lymphocytes 72. A 27-year-old woman complains of
- 11%, monocytes - 7%, ESR- 12 mm/hour. bleeding gums, nasal hemorrhages, multiple
What index should be determined additi- hematomas on the skin of her limbs and on
onally by a laboratory to make a diagnosis? the front of her torso, extreme general fati-
gue. Blood test: Hb- 64 g/l, erythrocytes -
2, 5 · 1012 /l, reticulocytes - 16%, platelets -
Крок 2 Medicine (англомовний варiант, iноземнi студенти) 2016 рiк 10

30 · 109 /l, ESR- 22 mm/hour. What approach adrenal glands, cranium X-ray. The tests
would be most efficient for treatment of this revealed no pathologies.The diagnosis of
pathology? exogenic constitutive obesity has been made.
What direction of therapy should be prioriti-
A. Splenectomy zed?
B. Dicynone (Etamsylate)
C. Platelet concentrate transfusion A. Reducing diet and exercise
D. Cytostatics B. Sanatorium-and-spa treatment
E. Group B vitamins C. Anorectic drugs
D. Dehydration therapy
73. A 23-year-old patient had taken 1 g of E. ”Fat-burning” methods
aspirin to treat acute respiratory infection.
After that he developed an asthmatic fit wi- 77. An 8-year-old child with a 3-year history
th labored expiration that was arrested by of diabetes was hospitalized in hyperglycemic
introduction of aminophylline. The patient’s coma. Specify the initial dose of insulin to be
medical history is not burdened with allergies. administered:
The patient has undergone two surgeries for
nasal polyposis in the past. What diagnosis is A. 0,1-0,2 U/kg of body weight per hour
most likely? B. 0,05 U/kg of body weight per hour
C. 0,2-0,3 U/kg of body weight per hour
A. Aspirin-induced asthma D. 0,3-0,4 U/kg of body weight per hour
B. Atopic bronchial asthma E. 0,4-0,5 U/kg of body weight per hour
C. Infectious allergic bronchial asthma
D. Exercise-induced asthma 78. A worker, who was involved in fire fi-
E. Symptomatic bronchospasm ghting inside the building that stored 2 kg
of mercury, has been delivered to a hospital
74. A 58-year-old woman undergoing with complaints of emotional expansiveness,
chemotherapy for her oncologic disorder has palpitations, excessive sweating, body tremor,
developed sore throat. Examination revealed heart pain. Within one day his condition
necrotic areas on the mucosa of the pharynx aggravated. Objectively: the skin is pale and
and tonsils. Many of her teeth are afflicted wi- moist. The patient is depressed. Permanent
th caries. In blood: neutrophilic granulocytes red dermographism, erethism, unstable BP
are practically absent against the background are observed. What drug is the serum in this
of leukopenia. Leukocytes are represented case?
mainly by lymphocytes and monocytes. What
disease can be suspected in the given case? A. Unithiol
B. Atropine sulfate
A. Agranulocitar tonsillitis C. Calcium tetacine
B. Lacunar tonsillitis D. Amyl nitrite
C. Pseudomembranous (Vincent’s) tonsillitis E. Dipyroxime
D. Syphilitic tonsillitis
E. Diphtheria 79. During meat testing Trichinella was
detected in diaphragm crura in one of the two
75. A patient complains of constant dull muscular tissue samples. What tactics should
pain in the perineum and suprapubic area, a doctor choose regarding this meat?
weak flow of urine, frequent difficult pai-
nful urination, nocturia. The patient has A. Technolgical disposal
been suffering from this condition for several B. Incineration
months, during which urination was becomi- C. Boiling under 1,5 atmosphere
ng increasingly difficult, and pain in the peri- D. Preservation in 10% salt solution
neum has developed. On rectal examinati- E. Freezing under -12o C
on: the prostate is enlarged (mainly its right 80. A 40-year-old patient was bitten by a stray
lobe), dense, asymmetrical, central fissure is dog an hour ago. On the left shin there is a
smoothed out, the right lobe is of stony densi- bite mark - the wound is 4x2x0,5 cm in size.
ty, painless, tuberous. What disease is it? What surgical aid would be most efficient in
A. Prostate cancer this case?
B. Prostate sclerosis A. Lavage with soapy water, retension sutures
C. Urolithiasis, prostatolith of the right lobe B. Aseptic dressing
D. Prostate tuberculosis C. Salve dressing
E. Chronic congestion prostatitis D. Blind suture
76. A child is 10 years old. The weight is E. Retension sutures
46 kg. Since birth the child has been gai- 81. A 30-year-old patient has been hospitali-
ning excessive weight. The parents are full- zed with diagnosis of intestinal obstruction.
bodied. The child has undergone the followi- The surgery revealed the obstruction of the
ng tests: carbohydrate tolerance, level of small intestine to be caused by a helminth
17-ketosteroids, blood electrolytes, US of ball. What kind of helminth is it?
Крок 2 Medicine (англомовний варiант, iноземнi студенти) 2016 рiк 11

A. Shigellosis
A. Ascaris B. Salmonellosis
B. Guinea worm C. Escherichiosis
C. Filariidae D. Intestinal amebiasis
D. Cysticercus E. Yersiniosis
E. Pinworm
86. The institutions that take part in medi-
82. Mother of a newborn suffers from chronic cal examinations include prevention and
pyelonephritis; she had a case of URTI before treatment facilities, medical board of Mini-
the delivery. The delivery is at term, for a stry of Defense, medical board of Ministry of
long period waters were not breaking. On Home Affairs, medico-social expert commi-
the 2nd day of life the child developed an ssions, forensic medical boards etc. What insti-
erythematic rash, later the rash developed tutions are responsible for temporary disabi-
into blisters approximately 1 cm in diameter lity examination?
filled with serous purulent substance. Ni-
kolsky’s symptom is positive. After the bli- A. Prevention and treatment facilities
sters had been lanced, erosions developed in B. Sanitary-and-prophylactic institutions
their place. The child is inert. The child’s body C. Medico-social expert commissions
temperature is subfebrile. What diagnosis is D. Medical boards of Ministry of Defense
most likely? E. Medical boards of Ministry of Home Affairs

A. Impetigo neonatorum 87. After a lengthy march an army regi-


B. Vesiculopustulosis ment has set camp for 3 days near a
C. Pseudofurunculosis settlement. Sanitary-hygienic investigation
D. Sepsis detected several water sources. Choose the
E. Ritter’s disease source that would satisfy the demands for
potable water the most under the given fi-
83. Three weeks after a case of acute tonsilli- eld conditions:
tis the patient is still weak, inert, subfebrile,
his retromaxillary lymph nodes are enlarged. A. Artesian well
Tonsils are flabby, stick together with arches, B. Brook
there are purulent plugs in lacunae. What is C. River
the most probable diagnosis? D. Rain water
E. Melt water
A. Chronic tonsillitis
B. Chronic pharyngitis 88. Clinical statistical investigation was
C. Acute lacunar tonsillitis performed to determine efficiency of a new
D. Paratonsillitis pharmacological preparation for patients wi-
E. Tonsillar tumour th ischemic heart disease. What parametric
test (coefficient) can be used to estimate reli-
84. The following indicators were calculated ability of the results?
to analyse population health and treatment
quality in a cardiological hospital: primary A. Student’s t-distribution
cardiovascular morbidity - 62%; total cardi- B. Sign test
ovascular morbidity - 483,55%; cardiovascular C. Matching factor
mortality - 10,9%; proportion of cardi- D. Wilcoxon signed-rank test
ovascular mortality within total mortality - E. Kolmogorov-Smirnov test
67,0%; primary disablement caused by cardi-
ovascular diseases - 16,2 per 10.000 populati- 89. A 30-year-old parturient woman was deli-
on. What indicator is an extensive value? vered to a maternity hospital with full-term
pregnancy. She complains of severe lanci-
A. Proportion of cardiovascular mortality nating pain in the uterus that started 1 hour
within total mortality ago, nausea, vomiting, cold sweat. Anamnesis
B. Primary cardiovascular morbidity states cesarean section 2 years ago. Uteri-
C. Total cardiovascular morbidity ne contractions stopped. Skin and mucous
D. Primary disablement caused by cardi- membranes are pale. Heart rate is 100/min.,
ovascular diseases BP is 90/60 mm Hg. Uterus has no clear margi-
E. Cardiovascular mortality ns, is sharply painful. No heartbeat can be
auscultated in the fetus. Moderate bloody
85. A 6-year-old child complains of frequent discharge from the uterus can be observed.
liquid stool and vomiting. On the 2nd day Uterus cervix is 4 cm open. Presenting part is
of disease the child presented with inertness, not visible. The most likely diagnosis is:
temperature rise up to 38, 2o C , Ps- 150/min.,
scaphoid abdomen, palpatory painful sigmoid
colon, defecation 10 times a day with liquid,
scarce stool with mucus and streaks of green.
What is the provisional diagnosis?
Крок 2 Medicine (англомовний варiант, iноземнi студенти) 2016 рiк 12

A. Uterine rupture
B. Initial uterine rupture A. Testicular feminization syndrome
C. Threatened uterine rupture B. Mayer-Rokitansky-Kuster-Hauser
D. Premature detachment of normally positi- syndrome
oned placenta C. Cushing’s syndrome
E. Compression of inferior pudendal vein D. Sheehan syndrome
E. Cushing’s disease
90. A parturient woman is 23 years old.
Internal obstetric examination shows the 94. A planner designs a heating system for a
uterine cervix to be completely open. Fetal pre-school educational establishment. The hi-
bladder is absent. Cephalic presentation is ghest air temperature should be in the followi-
observed in the plane of the small pelvic ng room:
outlet. Sagittal suture is at the longitudi-
nal section of the small pelvic outlet, small A. Game room of a nursery group
fontanel is situated closer to the uterus. What B. Common room of a preschool group
cephalic position will the newborn have duri- C. Bedroom of a nursery group
ng birth in this case? D. Bedroom of a preschool group
E. Gymnasium
A. Minor oblique lie
B. Longitudinal lie 95. A 27-year-old sexually active woman
C. Transverse lie complains of numerous vesicles on the right
D. Medium oblique lie sex lip, itch and burning. Eruptions regularly
E. Major oblique lie appear before menstruation and disappear
8-10 days later. What is the most likely di-
91. During the dynamic observation of a agnosis?
parturient woman in the second stage of labor
it was registered that the fetal heart rate A. Herpes simplex virus
decreased to 90-100/min. and did not normali- B. Bartholinitis
ze after contractions. Vaginal examination C. Primary syphilis
revealed the complete cervical dilatation, the D. Cytomegalovirus infection
fetal head filling the entire posterior surface E. Genital condylomata
of the pubic symphysis and sacral hollow;
the sagittal suture was in the anteroposteri- 96. A 35-year-old woman addressed a
or diameter of the pelvic outlet, the posterior gynecological in-patient department with
fontanelle was in front under the pubic arch. complaints of regular pains in her lower
What plan for further labour management abdomen, which increase during menstruati-
should be recommended? on, and dark-brown sticky discharge from
the genital tracts. On bimanual examinati-
A. Application of forceps minor on: the uterine body is slightly enlarged, the
B. Caesarean section appendages are not palpated. Mirror exami-
C. Episiotomy nation of the uterine cervix reveals bluish
D. Application of cavity forceps spots. What diagnosis is most likely?
E. Stimulation of labour activity through
intravenous injection of oxytocin A. Cervical endometriosis
B. Cervical erosion
92. An 8-year-old girl periodically has sudden C. Cervical polyp
short-term heart pains, sensation of chest D. Cervical cancer
compression, epigastric pain, dizziness, vomi- E. Cervical fibroid
ting. Objectively: the patient is pale, respi-
ratory rate - 40/min., jugular pulse is present. 97. A 10-year-old patient has a history of mild
Ps- 185/min., of poor volume. BP- 75/40 bronchial asthma. During a regular check-up
mm Hg. ECG taken during an attack shows the patient should be recommended:
ectopic P waves, QRS wave is not deformed. A. To avoid allergenic food
At the end of an attack a compensatory pause B. To avoid body tempering procedures
is observed. The most likely cause of the C. To avoid sports
attack is: D. To avoid spa treatment
A. Paroxysmal atrial tachycardia E. To avoid going to the seaside
B. Sinus tachycardia 98. A baby was born by a young smoker. The
C. Paroxysmal ventricular tachycardia labour was complicated by uterine inertia,
D. Complete AV-block difficult delivery of the baby’s head and
E. Atrial fibrillation shoulders. The baby’s Apgar score was 4.
93. A 16-year-old girl has primary Which of the following is a risk factor for a
amenorrhea, no pubic hair growth, normally spinal cord injury?
developed mammary glands; her genotype is
46 ХY; uterus and vagina are absent. What is
your diagnosis?
Крок 2 Medicine (англомовний варiант, iноземнi студенти) 2016 рiк 13

A. Difficult delivery of the head and shoulders te, depression. Objective examination reveals
B. Young age of the mother no pathologic changes of uterus and uterine
C. Pernicious habits appendages. What pathogenesis is characteri-
D. Uterine inertia stic of this disorder?
E. Chronic hypoxia
A. Decreased production of gonadotropin
99. An emergency situation at a chemical B. Hyperproduction of estrogen
plant caused acute occupational intoxicati- C. Hyperproduction of androgen
on. A doctor who revealed the case of ”acute D. Decreased production of progesterone
occupational disease (intoxication)” must E. Hyperproduction of prolactin
notify the following authority:
104. A patient in a clinical death condition is
A. Sanitary and epidemiological station being resuscitated through mouth-to-mouth
B. Plant administration artificial pulmonary ventilation and external
C. Trade union committee of the plant cardiac massage. A doctor noticed that air
D. Medical unit of the plant does not flow into the patient’s airways and
E. Ministry of Public Health of Ukraine his head and torso are positioned at the same
level. Why is artificial respiration not effective
100. A 27-year-old woman has been treated in in the given case?
a surgical department for pleural empyema
for 6 months. Multiple paracenteses of the A. Tongue retraction
pleural cavity were performed along with anti- B. Low breathing volume
bacterial treatment. The patient’s condition is C. External cardiac massage
slowly aggravating; attempts to fully stretch D. Probe is absent from stomach
the lung were unsuccessful. Choose the tacti- E. The patient’s mouth is too small
cs:
105. A 45-year-old patient with urolithiasis
A. Decortication of the lung had an attack of renal colic. What is the
B. Change antibiotics mechanism of the attack development?
C. Set constant active suction drain
D. Pulmonectomy A. Disturbed urine outflow from the kidney
E. Include hyperbaric oxygenation in the B. Increase in urine specific gravity
treatment C. Ureteric twists
D. Destruction of glomerules
101. A prematurely born girl is now 8 E. Renal artery spasm
months old. She has dyspnea, tachycardia,
hepatosplenomegaly, physical developmental 106. A 26-year-old woman has attended
lag, limb cyanosis. There is parasternal cardiac maternity center complaining of her inability
hump, auscultation revealed systolodiastolic to become pregnant despite 3 years of regular
murmur in the II intercostal space on the sex life. Examination revealed the followi-
left. BP is 90/0 mm Hg. What disease can be ng: increased body weight; male-type pubic
suspected? hair; excessive pilosis of thighs; ovaries are
dense and enlarged; basal body temperature
A. Patent ductus arteriosus is monophasic. The most likely diagnosis is:
B. Coarctation of aorta
C. Stenosis of aortal valve A. Ovaries sclerocystosis
D. Stenosis of pulmonary artery B. Inflammation of uterine appendages
E. Nonclosure of interventricular septum C. Adrenogenital syndrome
D. Premenstrual syndrome
102. Three days ago a boy underwent removal E. Gonadal dysgenesis
of a foreign body from under a nail plate. 2
days later he deeloped acute pulsating pain at 107. The process of open-cut mining requi-
the end of the nail bone which aggravated at res drilling and blasting operations, rock
pressing. Nail fold became hyperemic, body and ore excavation, transportation of ore
temperature rose up to 37, 5o C , there was a to fragmentation and sorting factories and
change in nail plate colour. What is the most transportation of barren rock to slag-heaps,
likely diagnosis? road building and maintenance, repair works.
What factor of production is most important
A. Subungual panaritium for miner’s health?
B. Erysipelas
C. Paronychia A. High content of dust in the air
D. Erysipeloid B. High content of explosion gas
E. Abscess C. Vibration
D. Noise
103. A 25-year-old woman complains of E. Adverse microclimate
menstruation retention lasting for 3 years.
The patient explains it by a difficult childbirth 108. A woman addressed a gynecologist
complicated with profuse hemorrhage, weight on the 20th day of puerperal period with
loss, brittleness and loss of hair, loss of appeti- complaints of pain in the left mammary gland,
Крок 2 Medicine (англомовний варiант, iноземнi студенти) 2016 рiк 14

puruent discharge from the nipple. Objecti- at the seaside. Objectively: face erythema,
vely: Ps- 120/min., body temperature is 39o C . edema of shin muscles. Heart sounds are
The left mammary gland is painful, larger than muffled, BP is 100/70 mm Hg. In blood:
the right one, the skin there is hyperemic; in ASAT activity is 0,95 millimole/h·l, ALAT-
the upper quadrant there is an infiltrate 10x15 1,3 micromole/h·l, aldolase - 9,2 IU/l, creati-
cm in size with soft center. Blood test: ESR- ne phosphokinase - 2,5 micromole Р/g·l. What
50 mm/hour, leukocytes - 15, 0 · 109 /l. What method of study would be most specific?
would be the treatment tactics?
A. Muscle biopsy
A. Transfer to a surgical department for surgi- B. ECG
cal treatment C. Echocardiogram
B. Refer to a gynecology department D. Electromyography
C. Refer to a postnatal department E. Determination of cortisol concentration in
D. Refer to a surgeon for conservative blood and urine
treatment
E. Lance the mammary gland abscess in a 113. A 14-year-old girl has fainted during a
maternity department meeting. The day before she complained of a
headache. The skin is pale, the limbs are cold,
109. The correlation between the service shallow breathing, heart sounds are muffled;
record and eosinophil concentration in blood heart rate is 51/min.; BP is 90/50 mm Hg. The
was studied in workers at dyeing shops of abdomen is soft. Meningeal symptoms are
textile factories. What index will be most negative. Make the provisional diagnosis:
informative for the analysis of this data?
A. Collapse
A. Correlation factor B. Unconsciousness
B. Student’s criterion C. Acute left ventricular failure
C. Standardized index D. Acute right ventricular failure
D. Fitting criterion E. Respiratory failure
E. Sign index
114. For the last 15 years a 48-year-old patient
110. A 3-month-old girl has rhinitis, dyspnea, has been working at the factory producing
dry cough. She has been sick for 2 synthetic resins. Lately he has been complai-
days. Objectively: pale skin, acrocyanosis, ning of significant general fatigue, headaches,
hypopnoe; breathing rate is 80/min.; over frequent urination (predominantly during the
the whole pulmonary surface there is day), red color of urine. What complication
vesiculotympanitic (bandbox) resonance of benzene nitrocompounds poisoning can be
observed with numerous bubbling crackles. suspected?
The most likely diagnosis is:
A. Malignant tumor of the urinary bladder
A. Acute bronchiolitis B. Chronic cystitis
B. Pneumonia C. Chronic prostatitis
C. Mucoviscidosis D. Acute glomerulonephritis
D. Foreign body in airways E. Chronic pyelonephritis
E. Acute bronchitis
115. A 60-year-old woman developed
111. A 24-year-old patient complains of gaini- weakness, vertigo, rapid fatigability during the
ng body mass and increased appetite. Objecti- last year. Recently there have been dyspnea
vely: built of hypersthenic type, body mass and paresthesia observed. Objectively: skin
index is 33,2 kg/m2 , waist circumference is and mucous membranes are pale and icteric.
100 cm. Waist to hips circumference ratio is Lingual papillae are smoothed out. Liver
0,95. What is the provisional diagnosis? and spleen are situated at the edge of costal
arch. Blood test: Hb- 70 g/l, erythrocytes -
A. Alimentary constitutive obesity, I stage, 1, 7·1012 /l, blood color index - 1,2, macrocytes.
android type What drug can be prescribed on pathogenetic
B. Itsenko-Cushing hypothalamic obesity, II grounds?
stage, gynoid type
C. Alimentary constitutive obesity, III stage, A. Vitamin B12
gynoid type B. Vitamin B6
D. Alimentary constitutive obesity, II stage, C. Ascorbic acid
android type D. Iron preparations
E. Itsenko-Cushing hypothalamic obesity, I E. Vitamin B1
stage, android type
116. In the morning upon waking a 65-year-
112. A 36 year old female patient complains old patient developed weakness in the right-
ofgeneral weakness, edemas of her face and side limbs, speech disorder, decreased sensi-
hands, rapid fatigability during walking, diffi- tivity of the left side of the body. On exami-
cult diglutition, cardiac irregularities. These nation: conscious, BP- 100/60 mm Hg, motor
symptoms developed 11 days after holiday aphasia, right-sided central hemiparesis and
Крок 2 Medicine (англомовний варiант, iноземнi студенти) 2016 рiк 15

hemihypalgesia. Make the preliminary di- mmol/l, potassium - 6,5 mmol/l, glomerular
agnosis: filtration rate - 10 ml/min. What tactics would
be leading in the patient’s treatment?
A. Ishemic stroke
B. Hemorrhagic stroke A. Hemodialysis
C. Encephalitis B. Antibacterial therapy
D. Brain tumor C. Sorbent agents
E. Subarachnoid hemorrhage D. Blood transfusion
E. Hypotensive therapy
117. The objective of a statistical study was to
find out the extent of seeking medical care by 121. In a rural health care area there is a case
the population. For this purpose 300 residents of child dying during the first month of life. To
of the area were interviewed. Information was analyse this situation, among other measures,
collected by means of a special questionnaire. an expert assessment of medical records is
What method of collecting information was performed. What medical document should
used by researchers? be considered first?
A. Anamnestic A. Child development history
B. Immediate registration B. Infant development history
C. Immediate examination C. Vaccination card
D. Doing extracts D. Outpatient medical record
E. - E. Child’s medical record
118. A 57-year-old woman complains of havi- 122. A region attended by a central regional
ng a sensation of esophageal compresion, hospital demonstrates increased hemorrhagic
palpitation, difficult breathing during eating stroke morbidity. Essential hypertension
solid food, occasional vomiting with a full morbidity, however, remains at the same level
mouth, ”wet pillow” sign at night for the last and is below the average level registered wi-
6 months. Objectively: body tempearture - thin the larger area. What managerial decision
39o C , height - 168 cm, weight - 72 kg, Ps- should be made in this case?
76/min., BP- 120/80 mm Hg. X-ray revealed
a considerable dilation of the esophagus and A. To design and implement measures for
its constriction in the cardial part. What early diagnostics of arterial hypertension
pathology is most likely to have caused B. To design and implement measures for
dysphagia in this patient? primary prevention of essential hypertension
C. To design and implement measures for
A. Achalasia cardiae secondary prevention of hypertension compli-
B. Primary esophagism cations
C. Hiatal hernia D. To design and implement measures for
D. Esophageal carcinoma professional development of medical workers
E. Reflux esophagitis E. To design and implement improved di-
spensary system for hypertensive patients
119. A 25-year-old man complains of pain
in the lower third of his left thigh, which 123. A 42-year-old man was delivered to a
occurs both with and without physical exerti- surgical in-patient department with complai-
on. Possibility of trauma is denied by the nts of icteric skin, pain in the right subcostal
patient. Objectively: skin colour is normal; area. Biochemical blood analysis: total bili-
pastosity and pain can be detected with deep rubin - 140 mcmol/l, direct bilirubin - 112
palpation; knee joint mobility is reduced. X- mcmol/l. On US: choledoch duct - 1,4 cm,
ray of distal femoral metaphysis shows an a concrement is detected in the distal area.
area of destruction and spicule. Blood test: Gallbladder is 40 cm, no concrements. What
immature cells, no signs of inflammation. The treatment tactics should be chosen?
most likely diagnosis is:
A. Endoscopic papillosphincterotomy
A. Osteogenic sarcoma B. Laparoscopic cholecystectomy
B. Osteitis fibrosa cystica C. Laparotomy with choledoch duct drain
C. Chronic osteomyelitis D. Laparotomy with cholecystectomy
D. Multiple myeloma E. Threatment in an infectious diseases hospi-
E. Marble-bone disease tal
120. A 30-year-old woman suffers from 124. 4 weeks after myocardial infarction a 56-
polycystic renal disease. She has been admi- year-old patient developed acute heart pain,
tted with signs of fatigue, thirst and nocturia. pronounced dyspnea. Objectively: the pati-
Diuresis is up to 1800 ml per day. BP is ent’s condition is extremely grave, there is
200/100 mm Hg. Blood test: erythrocytes - marked cyanosis of the face, swelling and
1, 8 · 109 /l, Hb- 68 g/l. Urine analysis: specific throbbing of the neck veins, peripheral pulse
gravity - 1005, leukocytes - 50-60, erythrocytes is absent, carotid artery pulse is rhythmic,
- 3-5 in the vision field, creatinine - 0,82 130/min., BP is 60/20 mm Hg. Auscultation of
Крок 2 Medicine (англомовний варiант, iноземнi студенти) 2016 рiк 16

the heart reveals extremely muffled sounds, ght shin and dorsal surface of foot. Objecti-
percussion reveals heart border extension in vely: weakness of the anterior tibial muscle,
both directions. What is the optimal treatment long extensor muscle of the right toes, short
tactis for this patient? extensor muscle of the right toes. Low Achi-
lles reflex on the right. Positive Lasegue’s si-
A. Pericardiocentesis and immediate gn. What examination method would be most
thoracotomy effective for specification of the diagnosis of
B. Oxygen inhalation L5 root discogenic compression?
C. Puncture of the pleural cavity on the left
D. Conservative treatment, infusion of A. Magnetic resonance scan
adrenomimetics B. Spinal column X-ray
E. Pleural cavity drainage C. Electromyography
D. Angiography
125. A 45-year-old man was delivered to E. Lumbar puncture
a hospital with complaints of vomiting wi-
th streaks of blood, loss of weight. On 129. A 23-year-old female patient has been
esophagofiberscopy a cauliflower-shaped suffering from mental disorder since the
mucosal growth was detected in the abdomi- age of 18, the course of disorder has no
nal esophagus. The mucosa there bleeds on remission periods. At hospital the patient
contact. What preliminary diagnosis can be mostly presents with non-purposeful fooli-
made? sh excitation: she makes stereotypic gri-
maces, exposes herself, publically masturbati-
A. Esophageal tumor ng, loudly laughs, repeating stereotypical
B. Barrett esophagus obscene shouts. The patient should be prescri-
C. Abdominal esophagitis bed:
D. Esophageal diverticulum
E. Esophageal achalasia A. Neuroleptics
B. Antidepressants
126. A 35-year-old man complains of intense C. Tranquilizers
lumbar pain irradiating to the inguinal area, D. Nootropics
external genitalia, thigh; frequent urination, E. Mood stabilizers
chill, nausea, vomiting. Objectively: positive
Pasternatsky’s symptom (costovertebral angle 130. During preventive ultrasound scan of
tenderness). Urine analysis revealed that abdomen performed during regular check-
RBCs and WBCs covered the total field of up in a school the following was revealed in
microscope; the urine exhibited high protein an 11-year-old student of the 5th grade: the
concentration. These clinical presentations left kidney is 3 cm below the normal posi-
were most likely caused by the following tion, its shape, size and structure are within
pathology: the norm, the contralateral kidney cannot be
observed at its proper place. The preliminary
A. Urolithiasis, renal colic diagnosis is as follows: congenital anomaly of
B. Cholelithiasis, biliary colic renal development, dystopic left kidney, right
C. Renal infarct kidney is absent or pelvic dystopic. What X-
D. Intestinal obstruction ray method would be required for making the
E. Osteochondrosis, acute radicular syndrome final diagnosis and determining the functional
capacity of both kidneys?
127. A 20-year-old patient complains of pain
in the left lumbar region, arterial pressure rise A. Renal dynamic scintigraphy
up to 160/110 mm Hg. US revealed that the B. Radioimmunoassay
structure and size of the right kidney were wi- C. Radionuclide renography
thin age norms, there were signs of 3 degree D. Thermography
hydronephrotic transformation of the left ki- E. Excretory urography
dney. Doppler examination revealed an addi-
tional artery running to the lower pole of the 131. A young woman suffering from
kidney. Excretory urogram shows a narrowing seborrhea oleosa has numerous light-brown
in the region of ureteropelvic junction. Speci- and white spots on the skin of her torso
fy the treatment tactics: and shoulders. The spots have clear margi-
ns, branny desquamation, no itching. What
A. Surgical intervention provisional diagnosis can be made?
B. Administration of spasmolytics
C. Administration of ACE inhibitors A. Pityriasis versicolor
D. Kidney catheterization B. Torso dermatophytosis
E. Administration of β -blockers C. Seborrheic dermatitis
D. Pityriasis rosea
128. As a result of load lifting a 68-year-old E. Vitiligo
woman developed acute pain in the lumbar
region, in a buttock, posterolateral surface 132. In a traffic accident two persons died.
of her right thigh, external surface of the ri- An appointed forensic medical expert was
Крок 2 Medicine (англомовний варiант, iноземнi студенти) 2016 рiк 17

called on duty to another accident locati- A. Hypothyroid


on; therefore, according to the crininal law B. Hypo-ovarian
in force in Ukraine, an investigator called in C. Hypothalamic-pituitary
the following specialist for examination of the D. Alimentary and constitutive
location and bodies: E. Hypercorticoid

A. Nearest available medical doctor 136. A 10 week pregnant woman was admi-
B. Medical assistant tted to a hospital for recurrent pain in the
C. Surgical nurse lower abdomen, bloody discharges from the
D. Nearest available dispensing chemist genital tracts. The problems developed after
E. Nearest available dental mechanic a case of URTI. The woman was registered
for antenatal care. Speculum examination
133. A patient is 28 years old. He has been revealed cyanosis of vaginal mucosa, clean
suffering from mental disorder since he was cervix, open cervical canal discharging blood
22. His current condition has changed acutely: and blood clots; the lower pole of the gestati-
for 3 days the patient has been refusing to onal sac was visible. What tactics should be
leave his home. He claims that there is a chosen?
”telepathy” occurring between him and other
people, through which he receives ”thoughts A. Curettage of the uterus
of strangers” and transmits his own thoughts B. Pregnancy maintenance therapy
for everyone to hear. He thinks his thoughts C. Expectant management, surveillance
and actions are manipulated through this D. Hysterectomy
”telepathy”. Make the preliminary diagnosis: E. Antiviral therapy

A. Paranoid schizophrenia 137. A patient with fibromyoma of uterus si-


B. Depressive episode zed up to 8-9 weeks of pregnancy consulted
C. Manic episode a gynaecologist about acute pain in the lower
D. Organic delirium abdomen. Examination revealed pronounced
E. Acute reaction to stress positive symptoms of peritoneal irritation, hi-
gh leukocytosis. Vaginal examination revealed
134. A 40-year-old patient is registered in a that the uterus was enlarged correspondi-
narcological dispensary. Somatically: skin is ng to 9 weeks of pregnancy due to the fi-
dramatically hyperemic, sclera are injected, bromatous nodes, one of which was mobile
hyperhidrosis is present. BP- 140/100 mm and extremely painful. Appendages were not
Hg, heart rate - 100/min. Mental state: palpable. There were moderate mucous di-
autopsychic orientation is intact, allopsychic scharges. What is the optimal treatment tacti-
orientation is distorted. The patient presents cs?
with motor anxiety. There is an expression
of fear on his face. He refuses to talk about A. Urgent surgery (laparotomy)
his problems and asks to release him immedi- B. Surveillance and spasmolytic therapy
ately, because he ”may be killed”. This state C. Fractional diagnostic curettage of the uteri-
developed in a day after one of his regular ne cavity
drinking bouts. What is your provisional di- D. Surgical laparoscopy
agnosis? E. Surveillance and antibacterial therapy
A. Delirium tremens 138. A 4-month-old child was admitted to
B. Organic delirium a surgical department 8 hours after the first
C. Paranoia attack of anxiety. The attacks happen every
D. Alcoholic hallucinosis 10 minutes and last for 2-3 minutes, vomi-
E. Alcoholic paranoid ting occurred once. Objectively: the child’s
condition is grave. Abdomen is soft, palpation
135. A 35-year-old female patient has gai- reveals a tumor-like formation in the right ili-
ned 20 kg weight within a year with the ac area. After rectal examination the doctor’s
normal diet. She complains of chill, sleepi- finger was stained with blood. What is the
ness, dyspnea. The patient’s mother and sister most probable diagnosis?
are corpulent. Objectively: height - 160 cm,
weight - 92 kg, BMI- 35,9. Obesity is uniform, A. Ileocecal invagination
there are no striae. The face is amimic. The B. Gastrointestinal haemorrhage
skin is dry. The tongue is thickened. Heart C. Wilm’s tumour
sounds are muffled. Heart rate - 56/min., D. Helminthic invasion
BP- 140/100 mm Hg. The patient has consti- E. Pylorostenosis
pations, amenorrhea for 5 months. TSH- 28
mkME/l (normal rate - 0,32-5). Craniogram 139. A woman has focal encephalitis in the
shows no pathology. What is the etiology of anamnesis. Her spatial orientation is not di-
obesity in this case? srupted. She has a sensation, as if ”everythi-
ng seems surreal: buildings are small, round
or distorted; trees are upside down; people
are very tall with thin limbs”. Determine the
Крок 2 Medicine (англомовний варiант, iноземнi студенти) 2016 рiк 18

psychopathologic syndrome: A. Streptoderma


B. Herpes
A. Derealization C. Atopic cheilitis
B. Depersonalization D. Dermatitis
C. Hallucinatory E. Eczema
D. Oneiric
E. Cenestopathic 144. After a contact with chemicals a plant
worker has suddenly developed stridor, voi-
140. A 56-year-old woman complains of itchi- ce hoarseness, barking cough, progressi-
ng skin of her torso, constant nausea, consti- ng dyspnea. Objective examination reveals
pation, sensation of heaviness and pain in acrocyanosis. What is your provisional di-
the right subcostal area, extreme general fati- agnosis?
gue. The patient suffers from biliary cirrhosis.
The skin is pale icteric. The abdomen is soft, A. Laryngeal edema
the liver protrudes 2,0 cm from under the B. Laryngeal carcinoma
margin of the right costal arch, sensitive on C. PATE
palpation. Biochemical investigation: total bi- D. Pulmonary atelectasis
lirubin - 142,0 mcmol/l, conjugated bilirubin E. Pneumothorax
- 139,0 mcmol/l, alanine aminotransferase -
0,98 mmol/hour·l, aspartate aminotransferase 145. An 18-year-old woman complains of
- 0,82 mmol/hour·l, alkaline phosphatase - 8,7 pains in her lower abdomen, purulent di-
mmol/hour·l. What drug should be prescribed scharge from the vagina, temperature rise
in the first place? up to 37, 8o C . Anamnesis states that she
had random sexual contact the day before
A. Ursodeoxycholic acid the signs appeared. She was diagnosed wi-
B. Sirepar th acute bilateral adnexitis. On additional
C. Allochol examination: leukocytes in the all field of visi-
D. Essentiale forte (Phospholipides) on, bacteria, diplococci with intracellular and
E. Livolin forte extracellular position. What agent is most li-
kely in the given case?
141. A 40-year-old female patient complai-
ns of having a bulge on the anterior surface A. Gonococcus
of neck for 5 years. Objectively: Ps- 72/min., B. Colibacillus
arterial pressure - 110/70 mm Hg, in the ri- C. Chlamydia
ght lobe of thyroid gland palpation reveals a D. Trichomonad
mobile node 4x2 cm in size, the left lobe is E. Staphylococcus
not palpable, the basal metabolic rate is 6%.
What is the most likely diagnosis? 146. A 45-year-old man has been suffering
from duodenal ulcer disease for 5 years. He
A. Nodular euthyroid goiter complains of weakness, dizziness, dryness
B. Nodular hyperthyroid goiter of the skin. Objectively: the skin and vi-
C. Riedel’s thyroiditis sible mucosa are pale, chapped lips; heart
D. Mixed euthyroid goiter rate is 100/min., BP- 100/70 mm Hg, systolic
E. Median cervical cyst murmur at all points on heart auscultati-
on. All other internal organs are unchanged.
142. Estimation of community health level Fecal occult blood test is positive. Blood test:
involved analysis of a report on diseases regi- erythrocytes - 3, 1 · 1012 /l, Hb- 88 g/l, color
stered among the population of district under index - 0,7, leukocytes - 4, 6 · 109 /l, platelets -
charge (reporting form 12). What index is 350·109 /l, ESR- 21 mm/hour, anisocytosis, poi-
calculated based on this report? kilocythemia, serum iron - 9,5 mcmol/l. What
A. Common morbidity rate treatment tactics would you choose?
B. Index of pathological affection A. Iron preparations, balanced diet
C. Index of morbidity with temporary disabili- B. Concentrated red cells transfusion
ty C. Intramuscular introduction of 500 mkg of
D. Index of hospitalized morbidity cyanocobalamin
E. Index of basic non-epidemic morbidity D. Corticosteroids, cytostatics
143. A 32-year-old woman addressed a E. Ascorbic acid, calcium chloride
dermatologist with complaints of slightly 147. A 58-year-old patient complains of a
itching rashes in the mouth angles. She has headache in the occipital region, nausea,
been suffering from this condition for 3 choking, opplotentes. The presentations
days. Objectively: there are isolated small appeared after a physical exertion. Objecti-
phlyctenas and superficial erosions covered vely: the patient is excited. Face is hyperemic.
in honey-yellow scabs against the background Skin is pale. Heart sounds are regular, the
of slight hyperemia. Make the diagnosis: 2nd aortic sound is accentuated. BP- 240/120
mm Hg, HR- 92/min. Auscultation reveals
some fine moist crackles in the lower lungs.
Крок 2 Medicine (англомовний варiант, iноземнi студенти) 2016 рiк 19

Liver is not enlarged. ECG shows signs of apex beat to the left. What is the most likely
hypertrophy and left ventricular overload. diagnosis?
What is the most likely diagnosis?
A. Acute cardiac aneurysm
A. Complicated hypertensic crisis B. Recurrent myocardial infarction
B. Acute myocardial infarction, pulmonary C. Acute pericarditis
edema D. Cardiosclerotic aneurysm
C. Bronchial asthma exacerbation E. Pulmonary embolism
D. Uncomplicated hypertensic crisis
E. Community-acquired pneumonia 152. A 60-year-old patient complains of nearly
permanent sensation of heaviness and fullness
148. A 45-year-old patient (14-year-long work in the epigastrium, which increases after eati-
record as a house painter) upon the contact ng, foul-smelling eructation, occasional vomi-
with synthetic paint develops skin reddeni- ting with food consumed 1-2 days ago, wei-
ng, edema, severe itching and oozing lesi- ght loss. 12 years ago he was first found to
ons on her face. Symptoms disappear after have an ulcer of pyloric channel. The patient
the contact with this chemical substance has taken ranitidine for periodic hunger pains.
stops but even the smell of paint alone is The patient’s condition has been deteriorating
enough to make them reappear each time. over the last 3 months. Objectively: splashing
Each recurrence is characterised by increased sound in the epigastrium is present. What ki-
severity of symptoms. What provisional di- nd of complication is it?
agnosis can be made?
A. Pyloric stenosis
A. Professional eczema B. Penetration of gastric ulcer
B. Simple contact dermatitis C. Functional pyloric spasm
C. Allergic contact dermatitis D. Foreign body in the stomach (bezoar)
D. Urticaria E. Malignization of gastric ulcer
E. Toxicodermia
153. A 5-year-old child has body temperature
149. A 32-year-old woman addressed a increased up to febrile numbers, suffers from
maternity clinic with complaints of inferti- inertness, weakness. Examination revealed
lity that has been lasting for 7 years. hemorrhage on the skin of limbs and torso.
Her menstrual cycle occurs in two phases. Enlargement of cervical and axillary lymph
Hysterosalpingography reveals obstruction of nodes can be detected. The liver is 4 cm
the uterine tubes in the ampullar areas, an below the costal arch; the spleen is 6 cm
adhesive process in the small pelvis can be below the costal arch. Blood test: erythrocytes
observed. What treatment is most advisable - 2, 3 · 1012 /l, Hb- 60 g/l, platelets - 40 ·
in this case? 109 /l, leukocytes - 32, 8 · 109 /l, eosinophiles
- 1%, band neutrophiles - 1%, segmented
A. Laparoscopy neutrophiles - 12%, lymphocytes - 46%,
B. Laparotomy monocytes - 1%, blasts - 40%, Duke’s bleedi-
C. Tubectomy ng time is 9 min. What examination is
D. Adnexectomy necessary to make the diagnosis?
E. Hydrotubation
A. Myelogram
150. Survey radiograph of a 52-year-old B. Lymph nodes biopsy
worker of an agglomeration plant (28 years C. US of abdominal cavity
of experience, the concentration of metal dust D. Detection of hepatitis markers
is 22-37 mg/m3 ) shows mildly pronounced E. Investigation of platelets dynamic functions
interstitial fibrosis with diffused contrast well-
defined small nodular shadows. The patient 154. A child from primipregnancy was born in
has no complaints. Pulmonary function is a term labor and has body weight of 4000 g
not compromised. What is the provisional di- and body length of 57 cm. When born, he was
agnosis? nonresponsive to examination. Diagniosis is
diffuse. Heart rate is 80/min. What resuscitati-
A. Siderosis on measures should be taken?
B. Silicosis
C. Anthraco-silicatosis A. Begin ALV with mask
D. Silicatosis B. Introduce 100% oxygen
E. Anthracosis C. Intubate and begin ALV
D. Tactile stimulation
151. A week ago a 65-year-old patient E. Administer naloxone
suffered an acute myocardial infarction, his
general condition deteriorated: he complai- 155. A 15-year-old teenager has undergone
ns of dyspnea at rest, pronounced weakness. medical examination in military recruitment
Objectively: edema of the lower extremities, center. The following was revealed: interval
ascites is present. Heart borders are extended, systolic murmur at the cardiac apex, accent
paradoxical pulse is observed 2 cm from the of the II heart sound over the pulmonary
Крок 2 Medicine (англомовний варiант, iноземнi студенти) 2016 рiк 20

artery, tachycardia. What additional exami- is noted - lacrimation and itching. What provi-
nation method will be most informative for sional diagnosis can be suggested?
determining a diagnosis?
A. Reactive arthritis
A. Echocardiography B. Rheumatoid arthritis
B. Electrocardiography C. Seasonal pollinosis
C. X-ray D. Bacterial nonspecific urethral conjunctivitis
D. Phonocardiography E. URTI with conjunctiva and joints affected
E. Rheography
160. A 19-year-old patient complains of severe
156. A 64-year-old patient has been hospi- pain in axillary crease. Condition onset was a
talised with complaints of progressive jaundi- week ago after her swimming in a cold ri-
ce that developed over 3 weeks ago without ver and epilation. The next day painful ”boi-
pain syndrome, along with general weakness, l” appeared that was becoming larger every
loss of appetite. Objectively: temperature is day and became a plum-sized tumor. Upon
36, 8o C , heart rate is 78/min., abdomen is soft examination nodular conical growths joined
and painless, peritoneum irritation symptoms together are detected, the skin covering them
are not detected, palpation reveals sharply is bluish-red in color. Some nodules have fi-
enlarged tense gallbladder. What disease can stulous openings producing thick purulent
be characterised with these symptoms? mass. Body temperature is 38, 5o C , general
malaise. The most likely diagnosis is:
A. Cancer of pancreas head
B. Duodenal ulcer A. Hydradenitis
C. Acute cholecystitis B. Carbuncle
D. Chronic cholecystitis C. Cutaneous tuberculosis
E. Cholecystitis caused by lambliasis D. Necrotizing ulcerative trichophytosis
E. Pyoderma chancriformis
157. A 6-year-old girl attended a general
practitioner with her mother. The child 161. Annual report of an in-patient ward
complains of burning pain and itching in her presents data about the number of patient
external genitalia. The girl was taking antibi- days and the number of patients, who have
otics the day before due to her suffering from undergone treatment within a year. What
acute bronchitis. On examination: external work indicator of the in-patient ward can be
genitalia are swollen, hyperemic, there is whi- calculated based on these data?
te deposit accumulated in the folds. The most
likely diagnosis is: A. Average duration of in-patient treatment of
a patient
A. Candidal vulvovaginitis B. Bed occupancy rate
B. Trichomoniasis C. Hospital bed turnover rate
C. Nonspecific vulvitis D. Mortality
D. Helminthic invasion E. Efficient use of the hospital bed capacity
E. Herpes vulvitis
162. ECG revealed the following in a 10-
158. A patient with otopyosis is in sharply year-old child: sharp acceleration of the heart
deteriorating condition: he developed rate - 240/min., P wave overlaps with T wave
headache, vomiting, febrile temperature, and deforms it, moderate lengthening of PQ
general hyperesthesia. Meningeal symptoms interval, QRS complex is without changes.
and stagnant optic disks are observed. There What pathology does this child have?
are no focal symptoms. Liquor is turbid, blood
pressure is high, albuminocytological dissoci- A. Paroxysmal atrial tachycardia
ation occurs with neutrophils. What disease B. Atrial hypertrophy
can be suspected? C. Ventricular hypertrophy
D. WPW syndrome
A. Secondary purulent meningitis E. Extrasystole
B. Meningoencephalitis
C. Serous meningitis 163. A 54-year-old patient complains of
D. Primary purulent meningitis weakness, jaundice, itching skin. Disease
E. Subarachnoid hemorrhage onset was 1,5 months ago: fever up to 39o C
appeared at first, with progressive jaundice
159. A 25-year-old patient is not married and developed 2 weeks later. On hospitalisati-
has sexual relations with several partners. on jaundice was severely progressed. Liver
During the last 3 months he noticed small cannot be palpated. Gallbladder is enlarged
amount of mucoserous secretions produced and painless. Blood bilirubin is 190 mi-
from urethra. Subjectively: periodical itching cromole/l (accounting for direct bilirubin).
or burning pain in urethra. Two months ago Stool is acholic. What is the most likely jaundi-
pain in the knee join developed. Possibility of ce genesis in this patient?
trauma or exposure to cold is denied by the
patient. During the last week eye discomfort
Крок 2 Medicine (англомовний варiант, iноземнi студенти) 2016 рiк 21

A. Mechanical jaundice A. Neuroleptics with a sedative effect


B. Hepatocellular jaundice B. Antidepressants with an activating effect
C. Hemolytic jaundice C. Neuroleptics with an activating effect
D. Caroli syndrome D. Tranquilizers
E. Gilbert’s syndrome E. Antidepressants with a sedative effect
164. A 22-year-old patient complains of 8- 168. A 32-year-old man complains of pain in
month delay of menstruation. Anamnesis: the chest on the left, dyspnea, temperature
menarche since the age of 12,5. Since the rise up to 38, 0o C , slight cough. The disease
age of 18 menstruations are irregular. No onset was 2 weeks ago after overexposure to
pregnancies. Mammary glands have normal cold. He had suffered from bronchoadenitis
development; when the nipples are pressed, in his childhood. The affected side lags during
milk drops are discharged. On gynecologi- breating; percussion reveals dull sound with
cal examination: moderate uterus hypoplasia. oblique margin in the lower left lung, where
On hormonal examination: prolactin level breathing is absent. The right heart border is
exceeds the norm two times. On computed displaced outwards. Mantoux test with 2 TU
tomogram of the sellar region: a space- resulted in a papule 16 mm in size. What di-
occupying lesion 4 mm in diameter is agnosis is most likely?
detected. The most likely diagnosis is:
A. Tuberculous pleurisy
A. Pituitary tumor B. Central carcinoma of the left lung
B. Lactation amenorrhea C. Congestion pneumonia
C. Stein-Leventhal syndrome D. Thromboembolism of the pulmonary artery
D. Sheehan’s syndrome branches
E. Cushing’s disease E. Community-acquired pneumonia
165. A 2-year-old child in a satisfactory condi- 169. An emergency doctor has diagnosed a
tion periodically presents with moderate 32-year-old woman with generalized convulsi-
proteinuria, microhematuria. US results: the ve status epilepticus. The deterioration in the
left kidney is undetectable, the right one is patient’s condition is caused by a sudden gap
enlarged, there are signs of double pyelocali- in the epilepsy treatment. Specify the doctor’s
ceal system. What study is required to specify further tactics:
the diagnosis?
A. Hospitalization in the intensive care unit
A. Excretory urography B. Hospitalization in the department of
B. Micturating cystography neurology
C. Retrograde urography C. Hospitalization in the department of
D. Doppler study of renal vessels neurosurgery
E. Radioisotope renal scan D. Outpatient monitoring by a neuropathologi-
st
166. A 13-year-old girl complains of fatigabili- E. Outpatient monitoring by a neurosurgeon
ty, frequent headaches, cardialgia. Eight years
ago she had a case of pyelonephritis. Urine 170. A 19-year-old patient complains of
analyses periodically revealed leukocyturia. dyspnea during physical exertion. He often
The child has undergone no further treatment. has bronchitis and pneumonia. Cardiac
On examination: increased BP up to 150/100 murmur has been observed since his chi-
mm Hg. Ultrasound investigation revealed si- ldhood. On auscultation: there is splitting of
gnificant reduction of the right kidney. What the II heart sound over pulmonary arteria,
process is leading in arterial hypertension systolic murmur in the third intercostal space
pathogenesis in this case? near the left sternum margin. ECG test shows
right bundle-branch block. What is the provi-
A. Hyperactivity of renin-angiotensin system sional diagnosis?
B. Disruption of water-electrolytic balance
C. Disruption of renal circulation A. Interatrial septum defect
D. Hypersympathicotonia B. Open arterial duct
E. Increased cortisol level C. Coarctation of aorta
D. Aortic stenosis
167. A 26-year-old patient with affective bi- E. Mitral valve insufficiency
polar disorder has developed a condition
manifested by mood improvement, behavi- 171. A 75-year-old male patient complains
oural and sexual hyperactivity, verbosity, acti- of slight pain in the right iliac region. The
ve body language, reduced need for sleep. abdominal pain arose 6 days ago and was
Which of the following drugs are most effi- accompanied by nausea. Surgical examination
cient in this case? revealed moist tongue, Ps- 76/min., BP- 130/80
mm Hg. Abdomen was soft, slightly painful in
the right iliac region on deep palpation, the
symptoms of the peritoneum irritation were
doubtful. In blood: RBCs- 4, 0 · 1012 /l, Hb- 135
Крок 2 Medicine (англомовний варiант, iноземнi студенти) 2016 рiк 22

g/l, WBCs- 9, 5 · 109 /l, stab neutrophils - 5%, lymph nodes are observed. On mammary
segmentonuclear - 52%, lymphocytes - 38%, glands US: in the upper external quadrant of
monocytes - 5%, ESR- 20 mm/hour. Specify the right mammary gland there is a space-
the doctor’s further tactics: occupying lesion of increased echogenicity
21х18 mm in size. The most likely diagnosis
A. Emergency operation for acute appendicitis is:
B. Hospitalization, dynamic surveillance
C. Send the patient home A. Fibrous adenoma
D. Refer the patient to a district therapist B. Lacteal cyst
E. Administration of additional examination: C. Diffuse mastopathy
abdominal ultrasound, x-ray contrast study of D. Breast cancer
the gastrointestinal tract E. Mastitis
172. An 8-year-old boy has a 2-year history of 176. 2 weeks after labour a parturient
blotchy itchy rash appearing after eating ci- woman developed breast pain being observed
trus fruit. The first eruption occurred at the for 3 days. Examination revealed body
age of 6 months after the introduction of jui- temperature at the rate of 39o C , chills,
ces to the baby’s diet. Father has a history of weakness, hyperaemia, enlargement, pain and
bronchial asthma, mother - that of allergic rhi- deformity of the mammary gland. On palpati-
nitis. What is the most likely diagnosis? on the infiltrate was found to have an area of
softening and fluctuation. What is the most
A. Atopic dermatitis likely diagnosis?
B. Psoriasis
C. Pityriasis Rosea A. Infiltrative-purulent mastitis
D. Urticaria B. Phlegmonous mastitis
E. Quincke’s edema C. Lactostasis
D. Serous mastitis
173. A 25-year-old patient has been admi- E. Mastopathy
tted to the hospital with the following
problems: weakness, sweating, itching, wei- 177. A 48-year-old woman has been suffering
ght loss, enlarged submandibular, cervical, from chronic pancreatitis for the last 7 years.
axillary, inguinal lymph nodes. Objectively: Lately she has been noticing an increase in
hepatomegaly. Lymph node biopsy revealed daily feces with foul smell, abdominal di-
giant Berezovsky-Reed-Sternberg cells, stention, gurgling. The patient complains of
polymorphocellular granuloma composed of diarrhea, weakness, fatigability, loss of appeti-
lymphocytes, reticular cells, neutrophils, eosi- te, loss of weight. What syndrome can be
nophils, fibrous tissue, plasma cells. What is suspected in this case?
the most likely diagnosis?
A. Malabsorption
A. Lymphogranulomatosis B. Irritable colon
B. Lymph node tuberculosis C. Maldigestion
C. Lymphoreticulosarcoma D. Exudative enteropathy
D. Cancer metastases to lymph nodes E. Endocrine gland failure
E. Macofollicular reticulosis
178. A 37-year-old man suffers from attacks
174. An 8-year-old child was hospitalized of unconsciousness, dyspnea during physi-
for fever up to 39, 8o C , inertness, moderate cal exertion, periodical sensations of heart
headache, vomiting. Examination revealed rate disorder. Father of the patient died
meningeal symptoms. Lumbar puncture was suddenly at the age of 45. Objectively: heart
performed. The obtained fluid had raised rate is 90/min., BP is 140/90 mm Hg. On
opening pressure, it was transparent, with heart US: ejection fraction - 49%, significant
the cell count of 450 cells per 1 mcl (mai- myocardium thickening of the left ventri-
nly lymphocytes - 90%), glucose level of 2,6 cle and interventricular septum. What drug
mmol/l. What causative agent might have should be prescribed for the treatment?
caused the disease in the child?
A. Bisoprolol
A. Enterovirus B. Enalapril
B. Meningococcus C. Phenyhydinum (Nifedipine)
C. Koch’s bacillus D. Hydrochlorothiazide
D. Staphylococcus E. Furosemide
E. Pneumococcus
179. During a regular check-up of a 50-year-
175. A 25-year-old patient during self- old woman a tumor was detected in her ri-
examination detected a tumor in the upper ght mammary gland. The tumor is 5 cm in
external quadrant of the right mammary diameter, dense, without clear margins. The
gland. On palpation: painless, dense, mobi- skin over the tumor resembles lemon rind,
le growth 2 cm in diameter is detected in the the nipple is inverted. The lymph node can
mammary gland; no changes in the peripheral be palpated in the axillary region. What di-
Крок 2 Medicine (англомовний варiант, iноземнi студенти) 2016 рiк 23

agnosis is most likely? muffled, extrasystole is present, auscultati-


on reveals apical systolic murmur that is not
A. Breast cancer conducted to the left inguinal region. ESR is
B. Lacteal cyst 38 mm/hour. CRP is 2+, antistreptolysin O ti-
C. Diffuse mastopathy tre - 400. What is the most likely diagnosis?
D. Mastitis
E. Breast lipoma A. Acute rheumatic fever
B. Vegetative dysfunction
180. A 20-year-old woman complains of C. Non-rheumatic carditis
sensation of air shortage, lingering dull pain D. Juvenile rheumatoid arthritis
in the heart area, irritability. Objectively: E. Reactive arthritis
general condition is satisfactory, heart rate
lability, BP is 130/60 mm Hg. ECG reveals di- 184. An 18-year-old woman complains
sruption of repolarization proceses. The pati- of mammary glands swelling, headaches,
ent is diagnosed with somatoform autonomic tearfulness, abdominal distension occurring
dysfunction of cardial type. Specify the condi- the day before menstruation. The symptoms
tions of the patient treatment: disappear with the beginning of menstruation.
Menstruations are regular, last for 5-6 days wi-
A. Out-patient treatment th interval of 28 days in between. Gynecologi-
B. In-patient treatment in therapeutics cal examination revealed no changes of
department internal genitals. What is your diagnosis?
C. In-patient treatment in cardiology
department A. Premenstrual syndrome
D. In-patient treatment in cardiac surgery B. Sheehan syndrome
department C. Stein-Leventhal syndrome
E. In-patient treatment in psychiatric D. Asherman’s syndrome
department E. Adrenogenital syndrome
181. A 38-year-old man’s workplace is within 185. A 24-year-old patient had been delivered
the area of effect of ionizing radiation. Duri- to a thoracic department with a chest injury,
ng regular medical check-up he expresses a fracture of the IV, V, VI ribs on the right.
no complaints. Blood test: erythrocytes - Plan radiography shows the fluid level in the
4, 5 · 1012 /l, Нb- 80 g/l, leukocytes - 2, 8 · 109 /l, pleural cavity reaching the III rib on the right.
platelets - 30 · 109 /l. Can this person continue Puncture blood contained clots. What is the
to work with sources of ionizing radiation? optimal treatment tactics?

A. Work with radioactive substances and other A. Emergency thoracotomy


sources of radiation is contraindicated B. Pleural puncture
B. Work with radioactive substances is allowed C. Thoracentesis and thoracostomy
C. Only work with low-level radioactive D. Hemostatic therapy
substances is allowed E. Medical thoracoscopy
D. Work with radioactive substances can be 186. A 30-year-old man was delivered to
allowed after detailed medical examination a neurosurgical department with complai-
E. Work with radioactive substances is allowed nts of constant headaches, nausea, vomi-
only for limited periods of time ting, fever, weakness of the right-side li-
182. A maternity patient breastfeeding for 1,5 mbs. Anamnesis states that one month ago
weeks has attended a doctor. She considers the patient had a surgery for left-sided
the onset of her disease to be when proporti- suppurative otitis and mastoiditis. He has
onal breast engorgement occurred. Mammary been undergoing treatment in an ENT
glands are painful. Body temperature is department. Approximately 2 weeks ago
36, 6o C . Expression of breast milk is hindered. the temperature increased, and the patient
The most likely diagnosis is: developed headaches. Objectively: heart rate
- 98/min., BP- 140/90 mm Hg, temperature -
A. Lactostasis 38, 3o C . Neurologically pronounced stiff neck:
B. Infiltrative mastitis bilateral Kernig’s symptom, unsteadiness
C. Suppurative mastitis during the Romberg’s maneuver. Computer
D. Chronic cystic mastitis tomography of the brain revealed a three-
E. Gangrenous mastitis dimensional growth with a capsule in the left
hemisphere. Make the diagnosis:
183. An 11-year-old boy complains of general
weakness, fever up to 38, 2o C , pain and swelli- A. Cerebral abscess
ng of the knee joints, sensation of irregular B. Echinococcus
heartbeat. 3 weeks ago, the child had a case C. Hemorrhage
of tonsillitis. Knee joints are swollen, the D. Hydrocephalus
overlying skin and skin of the knee region E. Arnold-Chiari malformation
is reddened, local temperature is increased,
movements are limited. Heart sounds are 187. Examination of a Rh-negative pregnant
Крок 2 Medicine (англомовний варiант, iноземнi студенти) 2016 рiк 24

woman at 32 weeks of gestation revealed A. Mitral valve stenosis


a four-time rise of Rh-antibody titer wi- B. Mitral valve failure
thin 2 weeks, the titer was 1:64. In the first C. Aortic outflow stenosis
two pregnancies the patient had experienced D. Aortic valve failure
antenatal fetal death due to hemolytic di- E. Tricuspid valve stenosis
sease. What is the optimal tactics of pregnancy
management? 191. A 64-year-old woman has been suffering
from diabetes mellitus for the last 14 years.
A. Early delivery Approximately 3 days ago the skin on the
B. Delivery at 37 weeks of gestation distal phalanx of the I toe on the left foot
C. Screening for Rh-antibodies 2 weeks later became cold and bluish-black in color. Mild
and early delivery in case of further titer rise pain is observed in the affected area. Pulse on
D. Introduction of anti-Rh (D) the pedal arteries cannot be detected, pulse
immunoglobulin on the popliteal artery is retained. Glycemia
E. Ultrasound for signs of hemolytic disease of is 12,4 mmol/l. US scan: stenosis of the shin
the fetus arteries, collateral compensated blood flow.
Ankle-brachial pressure index is 0,7. Foot X-
188. A 74-year-old patient visited a urologi- ray: destruction of the distal phalanx of the
st with complaints of pain above the pubis I toe. Determine the grade of diabetic foot
and inability to urinate for 8 hours. At home according to Wagner:
he had taken antispasmodics and had a
warm bath but no improvement occurred. A. IV
Objectively: the abdomen is soft and pai- B. I
nful above the pubis; dullness of percussion C. II
sound is observed above the pubis. Murphy’s D. III
(Pasternatski’s) punch sign is negative on the E. V
both sides. What condition does the patient 192. A 4-month-old boy has been undergoi-
have? ng in-patient treatment for pneumocystic
A. Acute urinary retention pneumonia for 4 weeks. The diagnosis has
B. Paradoxal ischuria been made based on clinical signs, typi-
C. Chronic urinary retention cal X-ray presentation, presence of severe
D. Anuria hypoxemia, positive dynamics caused by
E. Oliguria intravenous introduction of Biseptol (Co-
trimoxazole). Anamnesis states that enzyme-
189. A 7-year-old child became ill again 2 linked immuno sorbent assay (ELISA)
weeks after a case of tonsillitis. There are the detected antibodies to HIV in the umbilical
following complaints: temperature rise up to blood. Polymerase chain reaction (PCR) was
38o C , hemorrhagic rash on the extremities, performed on the child at the ages of 1 month
enlargement of the ankle joints. Blood test: and 3 months, and proviral DNA was detected
hemoglobin is 120 g/l, platelets are 170 · 109 /l, in the child’s blood. Viral load and number of
ESR is 30 mm/hour. Urine test: proteinuria CD4+-lymphocytes was not measured. Make
up to 0,7 g/l, cylinders - 5-6 in the field of vi- the diagnosis:
sion, erythrocytes - 8-10 in the field of vision.
What mechanism of hemorrhagic syndrome is A. HIV/AIDS
present in the given case? B. Infectious mononucleosis
C. Pneumonia
A. Vessel wall damage caused by immune D. Tuberculosis
complexes E. Adenovirus infection
B. Platelet dysfunction
C. Suppression of hematopoietic stem cells 193. A 35-year-old patient undergoing
D. Decrease of adhesive-aggregative function treatment for heart disorder in cardiologi-
of platelets cal department has developed complaints
E. Vessel wall damage caused by bacteria of acute sudden pain in the epigastrium,
temperature rise up to 38, 3o C . Blumberg’s
190. A 33-year-old patient has developed and Razdolsky’s (abduction of femur) signs
dyspnea during physical exertion, palpitati- are positive. What necessitates surgical aid in
ons, disruptions of heart rate, swollen legs. the given case?
In the childhood the patient had a case of
acute rheumatic fever that required in-patient A. Progress as a surgical disease
treatment. There were no further requests B. Extent of congenital and acquired
for medical care. Objectively: heart rate is development disorders
92/min., rhythmic; BP is 110/70 mm Hg. At C. Severity of the concomitant pathology
the apex the I heart sound is increased, triple D. Urgent aid is required to save the patient’s
rhythm, diastolic murmur. What heart disease life
is most likely? E. Activity of the heart disorder
194. A 5-year-old girl was hospitalized with
Крок 2 Medicine (англомовний варiант, iноземнi студенти) 2016 рiк 25

complaints of pain and swelling in the right A. Doxazosin


knee joint, temperature rise up to 38, 4o C and B. Propranolol
a rash diagnosed as erythema annulare centri- C. Indapamide
fugum. The signs developed 3 days after the D. Amlodipine
recovery from a case of acute respiratory di- E. Captopril
sease. Name the etiotropic drug to be prescri-
bed: 198. On the 9th day after childbirth the
obstetric patient developed high fever up
A. Augmentin to 38o C . She complains of pain in the right
B. Methotrexate mammary gland. The examination revealed
C. Metypred (Methylprednisolone) the following: a sharply painful infiltrate can
D. Diclofenac sodium be palpated in the right mammary gland, the
E. Captopril skin over the infiltrate is red, subareolar area
and nipple are swollen and painful. What is
195. During medical examination of a group your diagnosis?
of children under 4 years carried out by a
pediatric team in one of the African countri- A. Abscess of the right mammary gland
es a set of similar pathological signs was B. Mastopathy
detected in some of the children. The si- C. Cancer of the right mammary gland
gns are as follows: growth inhibition, mental D. Serous mastitis
changes, muscle atrophy, swellings, changes E. Fibrous cystic degeneration of the right
in hair and skin pigmentation. These children mammary gland
were diagnosed with kwashiorkor. What food
products should be added to the diet to treat 199. A 16-year-old girl addressed a doctor wi-
this disorder? th complaints of fatigability and dizziness. On
heart auscultation: systolic murmur in the II
A. Fish, vegetables, cereals intercostal area along the breastbone edge
B. Milk, meat, vegetables on the left. ECG revealed signs of the right
C. Vegetables, fruit ventricular hypertrophy. X-ray revealed di-
D. Cereals, fruit, berries latation of the the pulmonary artery trunk,
E. Poultry, fruit, berries enlargement of the right heart. What heart di-
sorder is it?
196. An 8-year-old girl was diagnosed wi-
th signs of Morgagni-Adams-Stokes disease A. Pulmonary artery outflow stenosis
that developed against the background of the B. Fallot’s tetrad
III degree atrioventricular heart block. What C. Defect of the interatrial septum
drug should be introduced intravenously for D. Coarctation of aorta
emergency aid? E. Pulmonary artery valve failure

A. Atropine 200. A 32-year-old patient complains of


B. Prednisolone excessive appetite, excess weight, dyspnea
C. Dobutamine during physical exertion. There are fat deposi-
D. Digoxin ts in the area of abdomen and shoulder gi-
E. Potassium chloride rdle. The skin is pale-pink, adult male pattern
of hair distribution is observed on the torso,
197. A 62-year-old man addresed a urologist no stretch marks. Heart rate is 90/min., BP
with complaints of frequent urination at night is 120/80 mm Hg, body build index equals
(5-6 times per night), sensation of incomplete 35. Blood sugar is 4,9 mmol/l, cholesterol is
voiding of the urinary bladder, pain in the 6,2 mmol/l. On ophthalmoscopy: fundus of
lower abdomen, slow urination. Anamnesis: the eye without changes. What provisional di-
the II degree essential hypertension (peak agnosis can be made?
BP is 160/100 mm Hg). Current case: the II
degree enlargement of the prostate gland, A. Primary alimentary constitutive obesity,
PSA is 2,2 ng/ml. Select the drug suitable for android type
long-term therapy of the patient’s combined B. Primary alimentary constitutive obesity,
pathology: gynoid type
C. Secondary hypothalamic obesity
D. Secondary neuroendocrine obesity
E. Secondary endocrine hypo-ovarian obesity
INSTRUCTIONAL BOOK
Testing Board

TEST ITEMS FOR LICENSING EXAMINATION: KROK 2. MEDICINE.

Kyiv. Testing Board.


(English language).

Approved to print 15.03/№66. Paper size 60х84 1/8


Offset paper. Typeface. Times New Roman Cyr. Offset print.
Conditional print pages 24. Accounting publishing pages 28.
Issue. 1935 copies
List of abbreviations
List of abbreviations
ACTH Adrenocorticotrophic hormone
ADP Adenosine diphosphate
ALT, ALAT Alanin aminotranspherase
ALV Artificial lung ventilation
AP Arterial (blood) pressure
AST, AspAT Aspartat aminotranspherase
ATP Adenosine triphosphate
ВСG Bacillus Calmette-Guérin
BP Blood (arterial) pressure
CK Creatine kinase
CNS Central nervous system
СоА Coenzyme A
CRP C-reactive protein
CT Computed tomography
COX Cyclooxygenase
DIC Disseminated intravascular coagulation
DTP (DPT) Diphtheria-tetanus-pertussis vaccine
EDTA Ethylenediamine tetra-acetic acid
ELISA Enzyme-linked immunosorbent assay
ENT Ear, nose, and throat (as a department in a hospital)
EPR (ER) Endoplasmic reticulum
ESR Erythrocyte sedimentation rate
EV Enterovirus
FC Functional class
GABA Gamma aminobutyric acid
Hct, Ht Hematocrit
HDL High-density lipoproteins
HR Heart rate
IDL Intermediate-density lipoproteins
IU International unit
IUPAC International Union of Pure and Applied Chemistry
LDH Lactate dehydrogenase
LDL Low-density lipoproteins
LOX Lipoxygenase
MAO Monoamine oxidase
MRI Magnetic resonance imaging
NSAID Nonsteroidal anti-inflammatory drug
PE (PATE) Pulmonary embolism (Pulmonary artery thromboembolism)
PSA Prostate-specific antigen
RBC Red blood count
RR Respiratory rate
SES Sanitary-and-epidemiologic station
STD Sexually transmitted disease
STI Sexually transmitted infection
T/l Trillion/liter
TABT Typhoid-paratyphoid A and B plus tetanus toxoid vaccine
TMJ Temporomandibular joint
TSH Thyroid-stimulating hormone
TU Tuberculin unit
URTI Upper respiratory tract infection
V/f Vision field
VLDL Very-low-density lipoproteins
WBC White blood count
WPW Wolff-Parkinson-White syndrome

You might also like